+7 495 120-13-73 | 8 800 500-97-74

(для регионов бесплатно)

Содержание

Электродвижущая сила. | Объединение учителей Санкт-Петербурга

Электродвижущая сила.

Роль источника тока: разделить заряды за счет совершения работы сторонними силами. Любые силы, действующие на заряд, за исключением потенциальных сил электростатического происхождения (т. е. кулоновских) называютсторонними силами.

(Сторонние силы объясняются электромагнитным взаимодействием между электронами и ядрами)

ЭДС — энергетическая  характеристика источника. Это физическая величина, равная отношению работы, совершенной сторонни­ми силами при перемещении электрического заряда по замкнутой цепи, к этому заряду:

Измеряется в вольтах (В).

Еще одна характеристика источника — внутреннее сопротивление источника тока: r.

 

Закон Ома для полной цепи.

Энергетические преобразования в цепи:

— закон сохранения энергии

(А — работа сторонних сил; Авнеш.— работа тока на внешнем участке цепи сопротивлением RАвнутр.— работа тока на внутреннем сопротивлении источникаr.)

Закон ОмаСила тока в цепи постоянного тока прямо пропорциональна ЭДС источника тока и обратно пропорциональна полному сопротивлению электрической цепи.

Следствия:

 

1. Если R>>r, то ε=U. Измеряют e высокоомным вольтметром при разомкнутой внешней цепи.

2.Если R<<r, то ток   — максимальный ток для данной цепи (ток короткого замыкания).  Опасно, т.к.  — возрастает

e= U1+U2

3. На внутреннем участке цепи:   Aвнутр=U1q , на внешнем участке цепи: Aвнеш=U2q.

A=Aвнутр+ Aвнеш. Тогда: εq=U1q+U2q. Следовательно: ε= U1+U2

ЭДС источника тока равна сумме падений напряжений на внешнем и внутреннем   участках цепи.

 

4.  Если R

 растет, то I уменьшается.  — при уменьшении силы тока в цепи напряжение увеличивается!

 

5. Мощность: а) Полная..

б) Полезная. .

в) Теряемая. .

г) КПД   .

 

Соединение источников тока.

1. Последовательное соединение источников:  полная ЭДС цепи равнаалгебраической сумме ЭДС отдельных источников, полное внутреннее сопротивление равно сумме внутренних сопротивлений всех источников тока. Если все источники одинаковы и включены в одном направлении, то 

Тогда з-н Ома запишется в виде:

2. Параллельное соединение источников: один из источников (с наибольшейЭДС) работает как источник, остальные — как потребители (на этом принципе основана зарядка аккумулятора). Расчет по правилам Кирхгофа (см.).

Если все источники одинаковы , то закон Ома запишется в виде:.

Закон Ома для  неоднородного участка цепи .

—  знаки

«+» или «-« выбираются в зависимости от того, в одну или в противоположные стороны направлены токи создаваемые источником ЭДС и электрическим полем.

Правила Кирхгофа.

1. Алгебраическая сумма сил токов в каждом узле (точке разветвления) равна 0.    — следствие закона сохранения электрического заряда.

2. В любом замкнутом контуре цепи алгебраическая сумма произведений сил токов в отдельных участках на их сопротивления равна алгебраической сумме ЭДС источников в этих контурах.  

 — следствие закона Ома для неоднородного участка цепи.

Направление токов выбирают произвольно. Если после вычислений значение силы тока отрицательно, то направление противоположно.

Замкнутый контур обходят в одном направлении. Если направление обхода совпадает с направлением тока, то IR>0. Если при обходе приходят к «+» источника, то его ЭДС отрицательна.

В полученную систему уравнений должны входить все ЭДС и все сопротивления. Т.о. система должна состоять из одного уравнения для токов и  k-1 — го уравнения для ЭДС (k — количество замкнутых контуров).

 

Напряжение эдс формула. Внутреннее сопротивление — источник

Внутреннее сопротивление источников тока пренебрежимо мало.
Внутреннее сопротивление источника тока пренебрежимо мало.
Внутреннее сопротивление источника тока, рассчитанное по данной формуле, будет, строго говоря, действительно только для данного интервала нагрузок вследствие того, что поляризация не пропорциональна плотности тока.
Внутреннее сопротивление источника тока — сопротивление, которым обладает, источник тока. Это важная характеристика всякого источника тока, определяющая его внутреннее падение напряжения, напряжение, которое может создать источник на концах питаемой им цепи, и тот наибольший ток, который может дать источник при коротком замыкании.

Внутреннее сопротивление источника тока — сопротивление, которым обладает источник тока.
Внутренним сопротивлением источника тока, сопротивлениями соединительных проводов и контактов в ключах пренебречь.
Чему равно внутреннее сопротивление источника тока, ЭДС которого равна 30 В, если после включения внешней цепи сопротивлением 6 Ом напряжение на зажимах батареи стало равным 18 В.
Отсюда находим внутреннее сопротивление источника тока.
Здесь и далее внутренним сопротивлением источника тока и подводящих проводов следует пренебречь, если оно не задано в условии.
Здесь тэар при небольшом внутреннем сопротивлении источника тока и соответственно небольшом сопротивлении лампы rgK относительно невелико. Соответственно тзар, определяющееся в основном высоким сопротивлением RgK (получающимся в результате того, что при разряде потенциал сетки оказывается под отрицательным потенциалом относительно катода), становится во много раз больше, чем тзар и длина экспоненциального импульса на выходе (считая продолжительность для половины амплитуды) в несколько десятков раз превышает длительность импульса, по-данного на вход.
Определить электродвижущую силу и внутреннее сопротивление источника тока, если при одном положении движка реостата амперметр показывает 0 2 А, вольтметр — 1 8 В, а при другом положении движка — 0 4 Аи 1 6 В соответственно.
Обозначим через г — внутреннее сопротивление источника тока, через R — сопротивление каждого из вольтметров.
Ничем, так как внутреннее сопротивление источника тока бесконечно велико.
Сначала определим ЭДС и внутреннее сопротивление источника тока.

Напряжение на клеммах = напряжение без нагрузки — внутреннее сопротивление сила тока. Резюме: если источник тока или напряжения не загружен потребителем, ток не течет, и поэтому нет падения напряжения. На контактах источника питания можно измерять напряжение разомкнутой цепи. Когда к источнику питания подключена нагрузка, ток течет, а исходное напряжение разомкнутой цепи разделяется между сопротивлением нагрузки и внутренним сопротивлением источника питания.

Ток короткого замыкания: Наконец, существует третья возможность короткого замыкания контактов источника тока или напряжения. В то же время максимальный ток, который источник может обеспечить потоками. Он ограничен сопротивлением линии и внутренним сопротивлением источника напряжения. Важно: ток короткого замыкания, но напряжение короткого замыкания не существует теоретически. Существует ток короткого замыкания, который по определению является током, который течет, когда ток или источник напряжения закорочены.

Для определения ЭДС и внутреннего сопротивления источника тока к его выходу был подключен сначала резистор сопротивлением Д 2 Ом, затем — резистор сопротивлением Л2 4 Ом.
Наклон этих кривых определяется внутренним сопротивлением источника тока. В это понятие включается обычно как собственно омическое сопротивление, так и сопротивление, обусловленное поляризацией.

Здесь пренебрегают сопротивлением соединительных проводников и внутренним сопротивлением источника тока.
Для создания такого режима необходимо, чтобы внутреннее сопротивление источника тока было больше сопротивления базо-эмиг-терного перехода как в открытом, так и в закрытом состоянии. Чаще всего это условие выполняется при включении последовательно входу транзнсюра индуктивной катушки, которая одновременно является контурной катушкой.
При прохождении тока часть мощности выделяется на внутреннем сопротивлении источника тока.
К каким отрицательным последствиям приводит то, что внутреннее сопротивление источника тока дифференциального каскада имеет конечное значение.
Цепь из двух параллельных ветвей. U (в в течение t сек. равна.| Соотношения между единицами энергии. Мощность, передаваемая нагрузке, будет максимальной при раввщ ве внутреннего сопротивления источника тока и сопротивления нагрузки.
Зачастую серьезные недоразумения возникают у учащихся из-за неумения правильно учитывать влияние внутреннего сопротивления источников тока на режим работы всей электрической цепи. Ряд задач параграфа (например, 383, 385, 386, 392 — 395 и др.) посвящен специально выяснению этого вопроса, а также выяснению вопроса о выборе наиболее выгодных условий работы источников тока.
Кристаллы аммиаката цинка не-электропроводны, и образование этого соединения приводит к увеличению внутреннего сопротивления источника тока.
В любом замкнутом контуре (например, а ] 6, с алгебраическая сумма электродвижущих сил равна алгебраической сумме произведений величин токов на сопротивления отдельных участков цепи. Вычисляя сумму произведений токов на сопротивления отдельных участков цепи, следует учитывать также и внутренние сопротивления источников тока.
Если предположить, что емкость C0z пренебрежимо мала или включить ее в схему четырехполюсника Q, то внутреннее сопротивление источника тока / g можно считать действительным и равным У.
Получили, что максимальная мощность выделяется на нагрузке при условии, что величина внешнего сопротивления цепи R равна внутреннему сопротивлению источника тока.
Здесь под R понимается сопротивление всех резисторов, образующих цепь (сопротивление нагрузки), а под г — внутреннее сопротивление источника тока.

Существует также напряжение без нагрузки, которое по определению накладывается при отсутствии тока. Цель этой работы состояла в том, чтобы экспериментально определить электродвижущую силу, значение внутреннего сопротивления источника напряжения, используя в качестве параметра вариации восемь резисторов и проверить теорему максимальной передачи мощности от источника к резистивной нагрузке. При проверке условия максимальной передачи мощности наблюдалось, что это явление имело место, когда значения сопротивлений соответствовали, хотя даже с максимальной мощностью эффективность использования источника составляла всего 50%, поскольку половина генерируемой мощности рассеивается как тепло непосредственно в самом источнике.

Здесь под R понимается сопротивление всех резисторов, образующих цепь (сопротивление нагрузки), а под т — внутреннее сопротивление источника тока.
Механическая система и ее электрические модели (метод четырехполюсников. Как уже указывалось выше, внутреннее сопротивление источника напряжения (первая система аналогий) должно быть весьма малым, а внутреннее сопротивление источника тока (вторая система аналогий) — весьма большим, по сравнению с сопротивлением модели.
К положительным качествам рассматриваемого преобразователя следует отнести то, что в нем не предъявляется особо жестких требований к переходному сопротивлению ключей, так как величина их переходного сопротивления составляет лишь незначительную часть внутреннего сопротивления источника тока и не оказывает влияния на точность преобразования.
Итак, при последовательном включении п одинаковых источников тока электродвижущая сила образующейся батареи в п раз превышает электродвижущую силу отдельного источника тока, однако в этом случае складываются не только электродвижущие силы, но также и внутренние сопротивления источников тока. Такое включение является выгодным, когда внешнее сопротивление цепи весьма велико в сравнении с внутренним сопротивлением.
Следует отметить, что схема рис. 1 — 2 6 эквивалентна схеме рис. 1 — 1 а только в отношений энергии, выделяющейся в сопротивлении нагрузки R, и не эквивалентна ей в отношении энергии, выделяющейся во внутреннем сопротивлении источника тока.
Но Сумма разностей потенциалов замкнутой цепи равна нулю, сумма сопротивлений всех участков замкнутой цепи — это ее суммарное сопротивление, которое обычно записывают в виде двух слагаемых: R — внешнее (по отношению к источникам) сопротивление иг — внутреннее сопротивление источников тока.
Второй [ IMAGE ] Схема к примеру. В этом уравнении г и г % — внутренние сопротивления источни-ков тока е и е2 — на схеме не показаны; IR, IR2 и IRS — падения напряжения на внешних сопротивлениях цепи; / г, и / г2 — падения напряжений на внутренних сопротивлениях источников тока.
Внутреннее сопротивление источника тока может быть как чисто активным, так и реактивным.
Зависимость р / ро — отношения (выраженного в децибелах звукового давления на поверхности жесткого цилиндра (с высотой, равной его диаметру, куба, сферы к звуковому давлению, имевшему место в поле до их внесения, от отношения dA (или. /. — диаметра цилиндра или сферы (или ребра куба к длине волны. Параметр семейства кривых — угол Ф между осью цилиндра, куба, сферы и направлением прихода звука. При расчете микрофонных усилителей исходят из следующих соображений. Номинальное сопротивление микрофона является внутренним сопротивлением источника тока на входе усилителя, входное сопротивление усилителя — сопротивлением нагрузки микрофона.
В качестве источников тока в потенциометрии чаще всего применяют аккумуляторы или сухие элементы, значительно реже — стабилизированные источники постоянного тока. Современные потенциометры устроены таким образом, что внутреннее сопротивление источника тока не отражается на работе потенциометра. При работе с сухими батареями и аккумуляторами необходимо учитывать зависимость разрядного тока от времени, которая имеет минимальную крутизну через 10 — 15 мин после включения.
Распределение электрического напряжения вдоль обмотки сверхпроводящего магнита при образовании в нем нормальной зоны. На самом деле (рис. 9.2) высокий потенциал развивается внутри обмотки, где существует активная компонента напряжения, направленная навстречу индуктивной. Небольшая разность потенциалов между подводящими проводами обусловлена внутренним сопротивлением источника тока, который обычно автоматически отключается при переходе магнита в нормальное состояние. Но даже если это не произойдет, напряжение на источнике тока будет составлять всего лишь несколько вольт по сравнению с сотнями и, возможно, тысячами вольт в нормальной зоне. Поэтому напряжением источника можно пренебречь, но источник тока следует по возможности быстро отключить, чтобы не допустить длительного тепловыделения в обмотке и криостате.

Однако было отмечено, что результаты были удовлетворительными. Ключевые слова: резистор, сопротивление, электродвижущая сила, мощность, закон Ом, электроэнергия. Когда мы работаем с электрическими цепями, в некоторых случаях нам нужно устройство, которое поддерживает разность потенциалов между двумя терминалами, потому что, если бы не было этой разности потенциалов, не было бы электрического тока, протекающего по цепи, Для этого типа устройства он называется источником напряжения. Источником напряжения является устройство, которое при подключении к цепи подвергает носители заряда разности потенциалов, то есть обеспечивает энергию для движения через работу, выполняемую на носителях заряда.

Символом Rt на рис. 5.12, а обозначено внутреннее сопротивление источника тока.
Ключ, закорачивающий точку А на землю, с малым сопротивлением в открытом состоянии. Сопротивление открытого ключа обычно пренебрежимо мало по срав-нению с внутренним сопротивлением источника тока. Поэтому падение напряжения на ключе вызывает ничтожную погрешность.
Зависимость зарядного тока гео. На рис. 3 показана зависимость зарядного тока геометрической емкости от времени без учета токов абсорбции. Необходимо отметить, что спад тока в этом случае определяется внутренним сопротивлением источника тока, а не состоянием изоляции.
Хорошо, что при решении задачи Вы воспользовались методом эквивалентного активного двухполюсника. К сожалению, Вы ошиблись в определении значения сопротивления активного двухполюсника R3K: внутреннее сопротивление источника тока бесконечно велико, поэтому пассивный двухполюсник, к которому преобразуется схема рис. 6.13 а, при определении R3K будет содержать два резис-тивных элемента, соединенных последовательно.
К, так как в противном случае в выражении (5.1) должно быть учтено также напряжение непосредственно на входе усилителя. Вторым ограничивающим условием при выводе соотношения (5.1) является предположение о том, что внутреннее сопротивление источника тока весьма мало.
Таким образом, трансформатор изменяет величину сопротивления R в k2 раз. Этим широко пользуются при разработке различных электрических схем для согласования сопротивлений нагрузки с внутренним сопротивлением источников тока.
Основные типы однофазных трансформаторов.| Однофазные трансформаторы большой мощности. Таким образом, трансформатор изменяет величину сопротивления г в k2 раз. Этим свойством широко пользуются ьри разработке различных электрических схем для согласования сопротивлений нагрузки с внутренним сопротивлением источников тока.
Простей-шая электрическая цепь. Закон Ома справедлив не только для участка, но и для всей электрической цепи. В этом случае в значение R подставляется суммарное сопротивление всех элементов цепи, в том числе и внутреннее сопротивление источника тока. Однако при простейших расчетах цепей обычно пренебрегают сопротивлением соединительных проводников и внутренним сопротивлением источника тока.
Цепь постоянного тока. Напряжение, действующее во внешней электрической цепи источника тока, может быть представлено в виде суммы падений напряжения на отдельных элементах этой цепи. Но ведь ток, циркулирующий в цепи, протекает и через источник тока, который имеет свое сопротивление, называемое внутренним сопротивлением источника тока.

Именно факт выполнения работ на несущих нагрузках поддерживает разность потенциалов между терминалами. Эта «энергия», создаваемая источником напряжения, называется электродвижущей силой, также известной как ЭДС. Электродвижущая сила источника напряжения определяется как работа, выполняемая источником для передачи нагрузок от терминала нижней единицы заряда.

Источник напряжения может быть охарактеризован двумя типами: идеальным источником напряжения и фактическим источником напряжения. Идеальный источник напряжения — это тот, который не имеет сопротивления внутреннему перемещению нагрузок с одного терминала на другой. Разность потенциалов между выводами идеального источника равна его электродвижущей силе. В реальном источнике напряжения это не происходит. Внутри источника имеется несколько проводящих материалов, где каждый из них создает определенное сопротивление внутреннему перемещению нагрузок.

Допустим, есть простейшая электрическая замкнутая цепь, включающая в себя источник тока, например генератор, гальванический элемент или аккумулятор, и резистор, обладающий сопротивлением R. Поскольку ток в цепи нигде не прерывается, то и внутри источника он течет.

В такой ситуации можно сказать, что любой источник обладает некоторым внутренним сопротивлением, препятствующим току. Это внутреннее сопротивление характеризует источник тока и обозначается буквой r. Для или аккумулятора внутреннее сопротивление — это сопротивление раствора электролита и электродов, для генератора — сопротивление обмоток статора и т. д.

Рассматривая сопротивление, исходящее от всех проводников, встроенных в источник, мы можем определить их как единый резистор, эквивалентное сопротивление которого равно сумме сопротивлений всех проводников. Идеализация реального источника напряжения будет рассматривать его как идеальный источник напряжения, но с внутренним сопротивлением. Поэтому, когда реальный источник не подключен к цепи, разность потенциалов между выводами этого в точности равна значению его электродвижущей силы. С момента, когда источник подключен к цепи, он проводит ток, создавая разность потенциалов на выводах меньше, чем его электродвижущая сила.

Таким образом, источник тока характеризуется как величиной ЭДС, так и величиной собственного внутреннего сопротивления r – обе эти характеристики свидетельствуют о качестве источника.

Электростатические высоковольтные генераторы (как генератор Ван де Граафа или генератор Уимшурста), к примеру, отличаются огромной ЭДС измеряемой миллионами вольт, при этом их внутреннее сопротивление измеряется сотнями мегаом, потому они и непригодны для получения больших токов.

Через источник напряжения, а работа, выполняемая источником на зарядке, определяется уравнением, т.е. Согласно закону сохранения энергии работа, выполняемая источником, равна тепловой энергии, рассеиваемой в резисторах. Манипулируя уравнением, имеем. Разделив обе части уравнения на.

Тем не менее, разделив оба члена на. Целью эксперимента является определение значения внутреннего сопротивления источника напряжения и его электродвижущей силы. Для этого мы применяем метод, называемый методом сохранения энергии, чтобы вывести соотношение между проходящим через цепь электрическим током, электродвижущей силой источника и внутренними и внешними сопротивлениями.

Гальванические элементы (такие как батарейка) — напротив — имеют ЭДС порядка 1 вольта, хотя внутреннее сопротивление у них порядка долей или максимум — десятка Ом, и от гальванических элементов поэтому можно получать токи в единицы и десятки ампер.

Уравнение дает нам желаемое соотношение между. Тогда в реальном источнике разность потенциалов. Между двумя терминалами никогда не будет равняться силе. Электромотор, потому что, когда ток проходит через резисторы, система теряет потенциал. Таким образом, чем больше резистивных нагрузок в цепи, тем больше.

Учитывая взаимосвязь, предлагается определить. Мы манипулируем уравнением, чтобы оставить внешнее сопротивление в функции других членов. Тогда, поскольку уравнение прямой задается формулой. Рисунок 2 — Схема, используемая в эксперименте. В положении А в амперметре помещался мультиметр.

На данной схеме показан реальный источник с присоединенной нагрузкой. Здесь обозначены , его внутреннее сопротивление, а также сопротивление нагрузки. Согласно , ток в данной цепи будет равен:

Поскольку участок внешней цепи однороден, то из закона Ома можно найти напряжение на нагрузке:

Мы выбрали этот момент, потому что мы хотели бы определить падение напряжения в цепи в целом за счет изменения внешнего резистора. Итак, закрывая переключатель, измеряя. Обладая всеми упомянутыми материалами, схема была установлена ​​в соответствии с рисунком. Через показания, показанные в мультиметрах, значения напряжения и тока, связанные с используемым внешним резистором. Измерения проводились для восьми резисторов и, как правило, была построена таблица, связывающая падение напряжения и ток с каждым резистором.

Эти значения были отмечены и перечислены в таблице. Затем, по отношению к внешнему сопротивлению, падение напряжения и электрический ток, проходящие через резисторы, обратно пропорциональны. Необходимость показать эту связь между током и напряжением была только для изучения того, как эти величины ведут себя по мере изменения внешнего резистора. Таблица 1 — Связь между внешним сопротивлением, током и напряжением.

Выразив из первого уравнения сопротивление нагрузки, и подставив его значение во второе уравнение, получим зависимость напряжения на нагрузке от тока в замкнутой цепи:

В замкнутом контуре ЭДС равна сумме падений напряжений на элементах внешней цепи и на внутреннем сопротивлении самого источника. Зависимость напряжения на нагрузке от тока нагрузки в идеальном случае линейна.

Со значениями таблицы график был составлен, что связывает поведение падения напряжения в резисторах в зависимости от электрического тока, проходящего через схему. График 1 — Связь между напряжением и током. График 2 — Связь между падением напряжения и внешним сопротивлением.

Можно отметить, что это соотношение между падением напряжения и током в цепи обратно пропорционально. Таким образом, заменяя в, получаем. На графике можно найти экспериментальное определение значений электродвижущей силы и сопротивления. При этом необходимо было линеаризовать его, чтобы облегчить его анализ и интерпретацию. Для линеаризации была создана таблица, связывающая резисторы с обратным потоком, проходящим через них.

График это показывает, но экспериментальные данные на реальном резисторе (крестики возле графика) всегда отличаются от идеала:

Эксперименты и логика показывают, что при нулевом токе нагрузки напряжение на внешней цепи равно ЭДС источника, а при нулевом напряжении на нагрузке ток в цепи равен . Это свойство реальных цепей помогает экспериментально находить ЭДС и внутреннее сопротивление реальных источников.

Влияние внутреннего сопротивления на свойства двухполюсника

Эти значения были записаны в таблице. Электродвижущая сила. Таблица 4 — Сравнение значений, измеренных с экспериментальными значениями. Таблица 2 — Связь между внешним сопротивлением и обратным току. Кроме того, удалось проанализировать второй пункт, который должен был проверить теорему о состоянии передачи максимальной мощности от источника к резистивной нагрузке. Для этого был создан набор данных, связанный с мощностью, рассеиваемой каждым используемым резистором.

Со значениями таблицы график был нанесен на график, который показывает, по-видимому, линейное поведение сопротивления как функцию обратного тока. График 4 — Связь между внешним сопротивлением и обратным току. Основываясь на поведении графика, метод наименьших квадратов применялся для определения наилучшей линии, которая соответствует полученной точечной кривой. После применения метода и с помощью уравнений,, и, можно было определить значение электродвижущей силы и внутреннего сопротивления источника.

Экспериментальное нахождение внутреннего сопротивления

Чтобы экспериментально определить данные характеристики, строят график зависимости напряжения на нагрузке от величины тока, затем экстраполируют его до пересечения с осями.

В точке пересечения графика с остью напряжения находится значение ЭДС источника, а в точке пересечения с осью тока находится величина тока короткого замыкания. В итоге внутреннее сопротивление находится по формуле:

Для расчета мощности, рассеиваемой в каждом резисторе, использовалось уравнение. График 5 — Мощность, рассеиваемая во внешнем резисторе, в зависимости от внешних резисторов. График 6 — Мощность, рассеиваемая во внешнем резисторе, в зависимости от внешних резисторов.

Чтобы получить сравнение фактических значений с полученными. Кроме того, мощность, рассеиваемая во внутреннем резисторе, рассчитывалась как функция внешних резисторов. При этом таблица была собрана и, с этими значениями, была построена диаграмма следующим образом.

Экспериментально таблица была создана. Из таблицы был нанесен график. Обратите внимание, что поведение очень близко к поведению кривой графика, показывая удовлетворительные результаты, полученные до сих пор. График 7 — Рассеиваемая во внутреннем резисторе мощность в зависимости от внешних резисторов.

Развиваемая источником полезная мощность выделяется на нагрузке. График зависимости этой мощности от сопротивления нагрузки приведен на рисунке. Эта кривая начинается от пересечения осей координат в нулевой точке, затем возрастает до максимального значения мощности, после чего спадает до нуля при сопротивлении нагрузки равном бесконечности.

Из мощностей, рассеянных во внешних резисторах и внутреннем резисторе, можно определить полную мощность, рассеянную в цепи. Анализируя график, мы имеем, что мощность, рассеиваемая во внешних резисторах, известна как полезная мощность, то есть мощность, исходящая от источника, который фактически будет использоваться нагрузкой, находящейся в цепи. Мощность, рассеиваемая в резисторе, называется рассеиваемой мощностью, так как она заканчивается.

Преобразуется в тепловую энергию внутри самого источника и поэтому бесполезна другим элементам схемы. По-прежнему интерпретируя кривые графа, мы заметили, что кривые, образованные степенями. Таблица 8 — Общая мощность, рассеиваемая в резисторах. Рассеянные во внешних резисторах, имеют максимальную точку. На этом этапе происходит интересное явление, максимальное переключение мощности от источника к внешнему резистору. Чтобы определить эту точку, подставим уравнение в и вычислим его первую производную, приравняв ее к нулю.

Чтобы найти максимальное сопротивление нагрузки, при котором теоретически разовьется максимальная мощность при данном источнике, берется производная от формулы мощности по R и приравнивается к нулю. Максимальная мощность разовьется при сопротивлении внешней цепи, равном внутреннему сопротивлению источника:

Резистивными компонентами схемы, так как большая часть из них преобразуется в тепло в самом источнике, таким образом, «теряется». Когда значения резисторов равны. На равенство между рассеянными силами. При таком равенстве в диссипации предполагается, что суммарная мощность, которая заканчивается, рассеивается на 50% в каждом резисторе. Хотя мы имеем максимальную передачу мощности на внешний резистор, мы понимаем, что эффективность источника не является максимальной. Об этом легко убедиться, увидев равенство диссипации степеней, где половина генерируемой мощности рассеивалась в самом источнике в виде тепла.

Это положение о максимальной мощности при R = r, позволяет экспериментально найти внутреннее сопротивление источника, построив зависимость мощности, выделяемой на нагрузке, от величины сопротивления нагрузки. Найдя реальное, а не теоретическое, сопротивление нагрузки, обеспечивающее максимальную мощность, определяют реальное внутреннее сопротивление источника питания.

КПД источника тока показывает отношение максимальной выделяемой на нагрузке мощности к полной мощности, которую в данный момент развивает

Электродвижущая сила и конечное напряжение

Физика > ЭДС и конечное напряжение

 

Рассмотрите связь электродвижущей силы и конечного напряжения в электрической цепи: роль внутреннего сопротивления, разность потенциалов, формула и схемы.

Конечное напряжение (напряжение на выходе источника) основывается на электродвижущей силе и внутреннем сопротивлении.

Задача обучения

  • Переведите соотношение между ЭДС и конечным напряжением в формулу.

Основные пункты

  • Электродвижущая сила отображает разность потенциалов источника в момент, когда ток лишен движения.
  • Конечное напряжение – выход напряжения в устройстве, измеренное через клеммы.
  • Формула расчета: V = ЭДС — Ir.

Термины

  • Конечное напряжение – выход напряжения в устройстве, измеряемое на клеммах.
  • Электродвижущая сила (ЭДС) – напряжение, сформированное батареей или магнитной силой в соответствии с законом Фарадея.
  • Разность потенциалов – отличие в показателях потенциальной энергии между двумя точками в электрическом поле.

Если вы забудете выключить фары на машине, то со временем они потускнеют. Причина – разряд батареи. Почему же они просто не мигают при потере энергии? Постепенное затухание говорит о том, что конечное напряжение снижается по мере расхода заряда. Все дело в том, что у всех источников напряжения есть две главные части: источник электрической энергии и внутреннее сопротивление.

Электродвижущая сила

Все источники напряжения формируют разность потенциалов и могут отправлять ток при подключении к сопротивлению. Если брать небольшой масштаб, то из-за этого отличия создается электрическое поле, влияющее на заряды и вызывающее ток. Мы называем это отличие электродвижущей силой (ЭДС). Но не думайте, что перед нами «сила» в привычном понятии. Это особая разновидность разности потенциалов источника, когда ток лишен движения. Измеряется в вольтах.

ЭДС связана с источником разности потенциалов, но отличается от конечного напряжения при проходе тока. К примеру, напряжение на аккумуляторе будет меньшим, чем ЭДС. Если конечное напряжение можно вычислить без подачи тока, то оно приравнивается к ЭДС.

Конечное напряжение

Выход напряжения из устройства измеряется на клеммах и именуется конечным напряжением (V). Его вычисляют по формуле:

Любой источник напряжения обладает ЭДС, связанной с источником разности потенциалов. Также оно настраивается на сопротивление (r), связанное с конструкцией. Здесь видны выходные клеммы, по которым измеряют конечное напряжение (V). Так как V = ЭДС — Ir, конечное напряжение приравнивается к ЭДС, если ток отсутствует

V = ЭДС – Ir (r – внутреннее сопротивление, I – протекающий во время измерения ток).

I положителен, если ток течет от положительной клеммы. Чем больше ток, тем меньше конечное напряжение. Также конечное напряжение уменьшается при росте внутреннего сопротивления.


Закон Ома для полной цепи

1. Источник тока

При прохождении тока в проводнике выделяется некоторое количество теплоты. Согласно закону сохранения энергии при этом в электрическую цепь должна поступать энергия.

Может ли источником этой энергии быть электростатическое поле? Нет, не может, потому что при перемещении заряда вдоль всей цепи, то есть по замкнутой траектории, работа электростатического поля равна кулю.

Следовательно, для существования тока в замкнутой цепи в ней должен быть участок, на котором свободные заряды движутся против сил электростатического поля. Таким участком цепи является источник тока (рис. 59.1).

В источнике тока на свободные заряды действуют силы, которые имеют не электростатическую природу. Их называют сторонними силами. В результате действия сторонних сил происходит разделение зарядов: на одном полюсе источника тока накапливается положительный заряд, а на другом – отрицательный. Вследствие этого возникает электростатическое поле, которое движет свободные заряды в электрической цепи вне источника тока, то есть во внешней цепи.

В химических источниках тока сторонние силы имеют химическую природу. Например, если погрузить цинковый и медный электроды в серную кислоту, то положительные ионы цинка будут чаще покидать электрод, чем положительные ионы меди. В результате между медным и цинковым электродами возникнет разность потенциалов: потенциал медного электрода будет больше, чем цинкового. Медный электрод станет положительным полюсом источника тока, а цинковый – отрицательным.

В генераторах электростанций сторонними силами являются силы, действующие на свободные электроны в металле со стороны вихревого электрического поля, порождаемого переменным магнитным полем. Работа вихревого электрического поля по перемещению заряда вдоль замкнутого контура не равна нулю. Действие генераторов тока мы рассмотрим в курсе физики 11-го класса.

Электродвижущая сила источника тока

В источнике тока сторонние силы, перемещая свободные заряды против действия сил электростатического поля, совершают работу, которую мы обозначим Aстор.

Эта работа пропорциональна заряду q, который перемещается вдоль цепи за данный промежуток времени. Поэтому отношение работы сторонних сил к величине заряда не зависит ни от Aстор, ни от q. Следовательно, оно является характеристикой источника тока. Это отношение называют электродвижущей силой источника (ЭДС) и обозначают ξ:

ξ = Aстор/q.     (1)

(Это название не совсем удачно, потому что ЭДС – не «сила» в механическом смысле, а энергетическая характеристика источника.)

ЭДС, как и напряжение, измеряют в вольтах. Например, ЭДС батарейки составляет несколько вольт.

2. Закон Ома для полной цепи

Если сила тока в цепи равна I, то за время t по цепи проходит заряд q = It. Поэтому формулу (1) можно записать в виде

Aстор = ξIt.     (2)

При этом во внешней цепи сопротивлением R выделяется количество теплоты

Qвнеш = I2Rt,     (3)

а внутри источника тока выделяется количество теплоты

Qвнутр = I2rt,     (4)

где r – сопротивление источника, которое называют его внутренним сопротивлением.

Из закона сохранения энергии следует, что

Qвнеш + Qвнутр = Aстор.     (5)

? 1. Докажите, что из формул (2) – (5) следует:

I = ξ / (R + r).     (6)

Это соотношение называют законом Ома для полной цепи.

Сумму сопротивлений R + r называют полным сопротивлением цепи.

? 2. ЭДС источника тока 12 В, а его внутреннее сопротивление равно 2 Ом.
а) Чему равна сила тока в цепи, если сопротивление внешней цепи равно 4 Ом?
б) Какова максимально возможная сила тока в цепи? При каком сопротивлении внешней цепи это имеет место?

? 3. При внешнем сопротивлении 2 Ом сила тока в цепи равна 1,5 А, а при внешнем сопротивлении 4 Ом сила тока равна 1 А.
а) Чему равно внутреннее сопротивление источника?
б) Чему равна ЭДС источника?

Напряжение на полюсах источника

Закон Ома для полной цепи можно записать в виде

ξ = IR + Ir.     (7)

Первое слагаемое в этой формуле согласно закону Ома для участка цепи равно напряжению U на полюсах источника тока:

IR = U.

Поэтому формулу (7) можно записать в виде

U = ξ – Ir.     (8)

Формула (8) выражает зависимость напряжения U на полюсах источника тока от силы тока I в цепи.

Поставим опыт
Зависимость U(I) можно измерить на опыте, изменяя силу тока в цепи с помощью реостата (рис. 59.2, а, б). Красная пунктирная линия на схеме 59.2, б показывает, как идет ток в реостате. Например, если ползунок реостата, изображенного на рисунке 59,2, а, сдвинуть вправо, то сопротивление реостата увеличится, потому что увеличится длина обмотки, по которой идет ток.

? 4. На рисунке 59.3 изображен график зависимости U(I) для некоторого источника тока.

а) Чему равна ЭДС этого источника тока?
б) Чему равна наибольшая сила тока?
в) Чему равно внутреннее сопротивление источника тока?
г) Чему равно внешнее сопротивление, когда сила тока равна нулю?
д) Чему равно внешнее сопротивление, когда сила тока максимальна?
е) Чему равно внешнее сопротивление при I = 1,5 А?

Максимальное напряжение на полюсах источника равно ξ. Это имеет место при I = 0. Сила тока равна нулю, когда полюса источника разомкнуты (в этом случае внешнее сопротивление цепи является бесконечно большим).

Следовательно, напряжение между разомкнутыми полюсами источника тока равно ЭДС этого источника.

Минимальное же напряжение между полюсами источника равно нулю. Это имеет место при коротком замыкании, когда внешнее сопротивление R = 0. В этом случае сила тока максимальна. Ее называют силой тока короткого замыкания.

? 5. Покажите, что сила тока короткого замыкания выражается формулой

Iка = ξ/r.     (9)

Подсказка. Воспользуйтесь законом Ома для полной цепи.

Из формулы (9) видно, что при очень малом внутреннем сопротивлении источника (как, например, у автомобильного аккумулятора) сила тока короткого замыкания будет очень большой, что может вывести источник тока из строя.

? 6. Сила тока при коротком замыкании батарейки равна 2 А. Когда к батарейке подключили резистор сопротивлением 4 Ом, сила тока стала равной 1 А.
а) Как изменилось полное сопротивление цепи?
б) Чему равно внутреннее сопротивление батарейки?

Измерив напряжение на полюсах источника и силу тока в цепи при двух различных значениях сопротивления внешней цепи, можно найти ЭДС ξ и внутреннее сопротивление r источника тока. Это можно сделать графически и аналитически.

? 7. При силе тока в цепи 2 А напряжение на полюсах источника равно 8 В, а при силе тока 4 А напряжение на полюсах равно 4 В.
а) Постройте систему координат I, U и нанесите две точки графика зависимости U(I) согласно приведенным данным.
б) Проведите прямую через эти точки и отметьте точки пересечения этой прямой с осями координат. Используя этот график, найдите, чему равны ЭДС, сила тока короткого замыкания и внутреннее сопротивление источника тока.
в) Используя уравнение (8), составьте систему двух уравнений с двумя неизвестными ξ и r и решите ее.

3. КПД источника тока

Работу тока во внешней цепи называют полезной работой. Обозначим ее Aпол. Используя формулу для работы тока, получаем:

Aпол = I2Rt.

Поскольку источник обладает внутренним сопротивлением, полезная работа меньше работы сторонних сил, потому что часть работы сторонних сил расходуется на выделение в источнике тока количества теплоты I2rt. Поскольку

Aстор = I2Rt + I2rt,

получаем для отношения полезной работы к работе сторонних сил:

η = Aпол / Aстор = (I2Rt) / (I2Rt + I2rt) = R / (R + r).

Это отношение, выраженное в процентах, называют КПД источника тока.

? 8. При каком отношении внешнего сопротивления к внутреннему сопротивлению КПД источника тока равен: 50 %; 80 %? Почему случай, когда КПД источника тока равен 100 %, не представляет практического интереса?


Дополнительные вопросы и задания

9. На рисунке 59.4 изображена схема измерения зависимости напряжения U на полюсах источника тока от силы тока I. Амперметр и вольтметр считайте идеальными. Сопротивление всей обмотки реостата 16 Ом. При первом положении ползунка реостата показания приборов 3 А и 8 В, а при втором положении – 2 А и 12 В.

а) Как сдвинули ползунок реостата между первым и вторым измерениями – влево или вправо?
б) Чему равны ЭДС источника тока и его внутреннее сопротивление?
в) Каковы будут показания приборов, если ползунок реостата передвинуть в крайнее левое положение? в крайнее правое?

10. При силе тока 6 А мощность тока во внешней цепи равна 90 Вт, а при силе тока 2 А она равна 60 Вт.
а) Чему равна ЭДС источника тока?
б) Чему равно внутреннее сопротивление источника тока?
в) Чему равно напряжение на полюсах источника в первом и втором случаях?
г) Чему равен КПД источника тока в первом и втором случаях?

Эдс, разность потенциалов и напряжение — что это и в чем разница

В материалах по электротехнике и электронике часто можно встретить три физические величины, имеющие одну и ту же единицу измерения — Вольт: разность электрических потенциалов, электрическое напряжение и ЭДС — электродвижущая сила.

Чтобы раз и навсегда избавиться от путаницы в терминах, давайте разберемся, в чем же заключаются различия между этими тремя понятиями. Для этого подробно рассмотрим каждое из них по отдельности.

Разность электрических потенциалов

На сегодняшний день физикам известно, что источниками электрических полей являются электрические заряды или изменяющиеся магнитные поля. Когда же мы рассматриваем определенные точки А и В в электростатическом поле известной напряженности E, то можем тут же говорить и о разности электростатических потенциалов между двумя данными точками в текущий момент времени.

Эта разность потенциалов находится как интеграл электрической напряженности между точками А и В, расположенными в данном электрическом поле на определенном расстоянии друг от друга:

Практически такая характеристика как потенциал относится к одному электрическому заряду, который теоретически может быть неподвижно установлен в данную точку электростатического поля, и тогда величина электрического потенциала для этого заряда q будет равна отношению потенциальной энергии W (взаимодействия данного заряда с данным полем) к величине этого заряда:

Отсюда следует, что разность потенциалов оказывается численно равна отношению работы A (работа по сути — изменение потенциальной энергии заряда), совершаемой данным электростатическим полем при переносе рассматриваемого заряда q из точки поля 1 в точку поля 2, к величине данного пробного заряда q:

  • В этом и заключается практический смысл термина «разность потенциалов», применительно к электротехнике, электронике, и вообще — к электрическим явлениям.
  • И если мы говорим о какой-нибудь электрической цепи, то можем судить и о разности потенциалов между двумя точками такой цепи, если в ней в данный момент действует электростатическое поле, причем как раз потому, что рассматриваемые точки цепи будут находится одновременно и в электростатическом поле определенной напряженности.
  • Как было сказано выше, разность электрических потенциалов измеряется в вольтах (1 вольт = 1 Дж/1Кл).

Электростатическое поле — электрическое поле, создаваемое неподвижными электрическими зарядами.

Для того, чтобы электрические заряды были неподвижны, на них не должны действовать силы в тех местах, где эти заряды могли бы двигаться.

Но внутри проводников заряды могут свободно двигаться, поэтому при наличии электрического поля внутри проводников в них возникло бы движение зарядов (электрический ток).

Следовательно, заряды могут оставаться неподвижными только в том случае, если они создают такое поле, которое везде внутри проводников равно нулю, а на поверхности проводников направлено перпендикулярно к поверхности (т. к. иначе заряды двигались бы вдоль поверхности).

Для этого неподвижные заряды должны располагаться только по поверхности проводников и при том именно таким образом, чтобы электрическое поле внутри проводников было равно нулю, а на поверхности перпендикулярно к ней.

Все сказанное относится к случаю неподвижных зарядов. В случае движения зарядов, т. е. наличия токов в проводниках, в них должно существовать электрическое поле (т. к.

иначе не могли бы течь токи) и, следовательно, движущиеся заряды располагаются в проводниках, вообще говоря, не так, как неподвижные, и создают электрические поля, отличные по своей конфигурации от электростатического поля.

Но по своим свойствам электростатическое поле ничем не отличается от электрического поля движущихся зарядов.

Электрическое напряжение U

Теперь рассмотрим такое понятие как электрическое напряжение U между точками А и В в электрическом поле или в электрической цепи. Электрическим напряжением называется скалярная физическая величина, численно равная работе эффективного электрического поля (включая и сторонние поля!), совершаемой при переносе единичного электрического заряда из точки А в точку В.

Электрическое напряжение измеряется в вольтах, как и разность электрических потенциалов. В случае с напряжением принято считать, что перенос заряда не изменит распределения зарядов, являющихся источниками эффективного электростатического поля. И напряжение в этом случае будет складываться из работы электрических сил и работы сторонних сил.

Если сторонние силы отсутствуют, то работу совершит лишь потенциальное электрическое поле, и в этом случае электрическое напряжение между точками А и В цепи будет численно в точности равно разности потенциалов между данными точками, то есть отношению работы по переносу заряда из точки А в точку В к величине заряда q:

Однако в общем случае напряжение между точками A и B отличается от разности потенциалов между этими точками на работу сторонних сил по перемещению единичного положительного заряда:

Эту работу сторонних сил как раз и называют электродвижущей силой на данном участке цепи, сокращенно — ЭДС:

Электродвижущая сила — ЭДС

Электродвижущая сила — ЭДС так же, как и напряжение, в Международной системе единиц (СИ) измеряется в вольтах.

ЭДС является скалярной физической величиной, характеризующей работу непосредственно действующих сторонних сил (любых сил за исключением электростатических) в цепях постоянного или переменного тока. В частности, в замкнутой проводящей цепи ЭДС равна работе этих сил по перемещению единичного положительного заряда вдоль всего контура.

Здесь при необходимости вводят в рассмотрение электрическую напряженность сторонних сил Еex, являющуюся векторной физической величиной, равной отношению величины действующей на пробный электрический заряд сторонней силы к величине данного заряда. Тогда в замкнутом контуре L ЭДС будет равна:

Можно говорить об электродвижущей силе на любом участке электрической цепи. Это будет, по сути, удельная работа сторонних сил лишь на рассматриваемом ее участке.

ЭДС гальванического элемента, к примеру, есть ни что иное, как работа сторонних сил при перемещении единичного положительного заряда только внутри этого гальванического элемента, а именно — от одного его полюса к другому.

Работа сторонних сил не может быть выражена через разность потенциалов, так как сторонние силы непотенциальны и их работа зависит (!) от формы траектории. Так, например, работа сторонних сил при перемещении заряда между клеммами источника тока за пределами данного источника равна нулю.

Чем отличается ЭДС от напряжения: простое объяснение на примере

Многие люди (в то числе и некоторые электрики) путают понятие электродвижущей силы (ЭДС) и напряжения. Хотя эти понятия имеют отличия.

Несмотря на то, что они незначительные, не специалисту сложно в них разобраться. Не маловажную роль в этом играет единица измерения. Напряжение и ЭДС измеряются в одних единицах – Вольтах.

На этом отличия не заканчиваются, подробно обо всем мы рассказали в статье!

Что такое электродвижущая сила

Подробно этот вопрос мы рассмотрели в отдельной статье: https://samelectrik.ru/chto-takoe-eds-obyasnenie-prostymi-slovami.html

Под ЭДС понимается физическая величина, характеризующая работу каких-либо сторонних сил, находящихся в источниках питания постоянного или переменного тока.

При этом, если имеется замкнутый контур, то можно сказать, что ЭДС равна работе сил по перемещению положительного заряда к отрицательному по замкнутой цепи.

Или простыми словами, ЭДС источника тока представляет работу, необходимую для перемещения единичного заряда между полюсами.

При этом если источник тока имеющего бесконечную мощность, а внутреннее сопротивление будет отсутствовать (позиция А на рисунке), то ЭДС можно рассчитать по закону Ома для участка цепи, т.к. напряжение и электродвижущая сила в этом случае равны.

I=U/R,

где U – напряжение, а в рассмотренном примере — ЭДС.

Однако, реальный источник питания имеет конечное внутреннее сопротивление. Поэтому такой расчет нельзя применять на практике. В этом случае для определения ЭДС пользуются формулой для полной цепи.

I=E/(R+r),

где E (также обозначается как «ԑ») — ЭДС; R – сопротивление нагрузки, r – внутреннее сопротивление источника электропитания, I – ток в цепи.

Однако, эта формула не учитывает сопротивление проводников цепи. При этом необходимо понимать, что внутри источника постоянного тока и во внешней цепи, ток течет в разных направлениях. Разница заключается в том, что внутри элемента он течет от минуса к плюсу, то во внешней цепи от плюса к минусу.

Это наглядно представлено на ниже приведенном рисунке:

При этом электродвижущая сила измеряется вольтметром, в случае, когда нет нагрузки, т.е. источник питания работает в режиме холостого хода.

Чтобы найти ЭДС через напряжение и сопротивление нагрузки нужно найти внутреннее сопротивление источника питания, для этого измеряют напряжение дважды при разных токах нагрузки, после чего находят внутреннее сопротивление.

Ниже приведен порядок вычисления по формулам, далее R1, R2 — сопротивление нагрузки для первого и второго измерения соответственно, остальные величины аналогично, U1, U2 – напряжения источника на его зажимах под нагрузкой.

  • Итак, нам известен ток, тогда он равен:
  • I1=E/(R1+r)
  • I2=E/(R2+r)
  • При этом:
  • R1=U1/I1
  • R2=U2/I2
  • Если подставить в первые уравнения, то:
  • I1=E/( (U1/I1)+r)
  • I2=E/( (U2/I2)+r)
  • Теперь разделим левые и правые части друг на друга:
  • (I1/I2)= [E/( (U1/I1)+r)]/[E/( (U2/I2)+r)]
  • После вычисления относительно сопротивления источника тока получим:
  • r=(U1-U2)/(I1-I2)
  • Внутреннее сопротивление r:
  • r= (U1+U2)/I,
  • где U1, U2 — напряжение на зажимах источника при разном токе нагрузки, I — ток в цепи.
  • Тогда ЭДС равно:
  • E=I*(R+r) или E=U1+I1*r

Что такое напряжение

Электрическое напряжение (обозначается как U) – это физическая величина, которая отражает количественную характеристику работы электрического поля по переносу заряда из точки А в точку В.

Соответственно напряжение может быть между двумя точками цепи, но в отличии от ЭДС оно может быть между двумя выводами какого-то из элементов цепи.

Напомним, что ЭДС характеризует работу, выполненную сторонними силами, то есть работу самого источника тока или ЭДС по переносу заряда через всю цепь, а не на конкретном элементе.

Это определение можно выразить простым языком. Напряжение источников постоянного тока – это сила, которая перемещает свободные электроны от одного атома к другому в определенном направлении.

Для переменного тока используют следующие понятия:

  • мгновенное напряжение — это разность потенциалов между точками в данный промежуток времени;
  • амплитудное значение – представляет максимальную величину по модулю мгновенного значения напряжения за промежуток времени;
  • среднее значение – постоянная составляющая напряжения;
  • среднеквадратичное и средневыпрямленное.

Напряжение участка цепи зависит от материала проводника, сопротивления нагрузки и температуры. Так же как и электродвижущая сила измеряется в Вольтах.

Часто для понимания физического смысла напряжения, его сравнивают с водонапорной башней. Столб воды отождествляют с напряжением, а поток с током.

При этом столб воды в башне постепенно уменьшается, что характеризует понижение напряжения и уменьшения силы тока.

Так в чем же отличие

Для лучшего понимания, в чем состоит разница электродвижущей силы от напряжения, рассмотрим пример. Имеется источник электрической энергии бесконечной мощности, в котором отсутствует внутреннее сопротивление. В электрической цепи смонтирована нагрузка. В этом случае будет справедливо утверждение, что ЭДС и напряжение тождественно равны, т.е между этими понятиями отсутствует разница.

Однако, это идеальные условия, которые в реальной жизни не встречаются. Эти условия используют исключительно при расчетах. В реальной жизни учитывается внутреннее сопротивление источника питания. В этом случае ЭДС и напряжение имеют отличия.

На рисунке представлено, какая разница будет в значениях электродвижущей силы и напряжении в реальных условиях. Вышеприведенная формула закона Ома для полной цепи описывает все процессы. При разомкнутой цепи на клеммах батарейки будет значение 1,5 Вольта. Это значение ЭДС. Подключив нагрузку, в данном случае это лампочка, на ней будет напряжение 1 вольт.

Разница от идеального источника заключается в наличии внутреннего сопротивления источника питания. На этом сопротивлении и происходит падение напряжения. Эти процессы описывает закон Ома для полной цепи.

Если измерительный прибор на зажимах источника электроэнергии показывает значение 1,5 Вольта, это будет электродвижущая сила, но повторим, при условии отсутствия нагрузки.

При подключении нагрузки на клеммах будет заведомо меньшее значение. Это и есть напряжение.

Вывод

Из вышесказанного можно сделать вывод, что основная разница между ЭДС и напряжением состоит:

  1. Электродвижущая сила зависит от источника питания, а напряжение зависит от подключенной нагрузки и тока, протекающего по цепи.
  2. Электродвижущая сила это физическая величина, характеризующая работу сторонних сил неэлектрического происхождения, происходящих в цепях постоянного и переменного тока.
  3. Напряжение и ЭДС имеет единую единицу измерения – Вольт.
  4. U -величина физическая, равная работе эффективного электрического поля, производимой при переносе единичного пробного заряда из точки А в точку В.

Таким образом, кратко, если представить U в виде столба воды, то ЭДС можно представить что это насос, поддерживающий уровень воды на постоянном уровне. Надеемся, после прочтения статьи Вам стало понятно основное отличие!

Материалы по теме:

Чем отличается ЭДС от напряжения

Чем отличается ЭДС (электродвижущая сила) от напряжения? Рассмотрим сразу на конкретном примере. Берем батарейку, на которой написано 1,5 вольт. Подключаем к ней вольтметр, как показано на рисунке 1, чтобы проверить, действительно ли батарейка исправна.

Рисунок 1

Вольтметр показывает 1,5 В. Значит, батарейка исправна. Подключаем ее к маленькой лампочке. Лампочка светится. Теперь параллельно лампочке подключаем вольтметр, чтобы проверить: действительно ли на лампочку приходится 1,5 В. Получается схема, показанная на рисунке 2.

Рисунок 2

И тут оказывается, что вольтметр показывает, например, 1 В. Куда потрачены 0,5 В (которые разность между 1,5 В и 1 В)?

Дело в том, что любой реальный источник питания имеет внутреннее сопротивление (обозначается буквой r).

Оно во многих случаях снижает характеристики источников питания, но изготовить источник питания вообще без внутреннего сопротивления невозможно.

Поэтому нашу батарейку можно представить как идеальный источник питания и резистор, сопротивление которого соответствует внутреннему сопротивлению батарейки (рисунок 3).

  • Рисунок 3
  • Так вот, ЭДС в данном примере – это 1,5 В, Напряжение источника питания – 1 В, а разница 0,5 В была рассеяна на внутреннем сопротивлении источника питания.

ЭДС – это максимальное количество вольт, которое источник питания может выдать в цепь. Это постоянная для исправного источника питания величина. А напряжение источника питания зависит от того, что к нему подключено. (Здесь мы говорим только о тех типах источников питания, которые изучаются в рамках школьной программы).

  1. В нашем примере лампочка с сопротивлением R и резистор соединены последовательно, поэтому ток в цепи можно найти по формуле
  2. И тогда напряжение на лампочке равно

Получается, чем больше сопротивление лампочки, тем больше вольт приходится на нее, и тем меньше вольт бесполезно теряется в батарейке. Это касается не только лампочки и батарейки, но и любой цепи, состоящей из источника питания и нагрузки.

Чем больше сопротивление нагрузки, тем меньше разница между напряжением и ЭДС. Если сопротивление нагрузки очень большое, то напряжение практически равно ЭДС.

Сопротивление вольтметра всегда очень большое, поэтому в схеме на рисунке 1 он показал значение 1,5 В.

Пониманию смысла ЭДС мешает то, что в быту мы этот термин практически не употребляем. Мы говорим в магазине: «Дайте мне батарейку с напряжением 1,5 вольта», хотя правильно говорить: «Дайте мне батарейку с ЭДС 1,5 вольта». Но так уж повелось…

  • Похожая статья: чем отличается напряжение от потенциала.

Чем отличается ЭДС от напряжения

Интересно многие сразу поняли, в чем разница между ЭДС и напряжением? И никого не поправлял учитель (учительница) по физике, когда на практических занятиях говорил (-ла) о том, что мы подключаем именно источник ЭДС, а не напряжения? В большинстве случаев мы с вами путались, потому что и ЭДС, и напряжение измеряется в Вольтах. Так давайте все-таки разберемся, чем принципиально отличается ЭДС от напряжения.

Что такое ЭДС

Итак, для начала давайте разберемся, что такое ЭДС. Электродвижущая сила (ЭДС) — это такая физическая величина, которая характеризует работу сторонних (не потенциальных) сил в источниках переменного либо же постоянного тока.

В замкнутой цепи ЭДС — это работа сил, совершаемая для перемещения единичного заряда вдоль всего контура.

Из выше представленного определения вытекает следующее: источниками ЭДС являются силы, которые не имеют прямое отношение к электростатике, но при этом они являются силами, которые создают движение заряда в замкнутой электрической цепочке.

yandex.ru

Например, при механическом вращении обмотки ротора в электромагнитном поле, в ней будет формироваться индукционная ЭДС. При этом формирование ЭДС будет проходить в каждом витке отдельно, но при этом электродвижущая сила соседних витков будет складываться, и на выходе мы будем иметь сумму ЭДС всех витков.

Если посмотреть на аккумуляторные батареи, то в них источником ЭДС является химическая реакция.

Кроме этого источниками могут выступать так называемые элементы Пельтье, в которых ЭДС образуется при термическом нагреве.

Пьезоэффект (когда при механическом воздействии на материал на его концах образуется разность потенциалов) также относится к источникам ЭДС. Впрочем, как и фотоэффект.

yandex.ru

  • Из выше представленных примеров видно, что, применяя различные материалы и способы их взаимодействия, можно получить ЭДС, способную организовать упорядоченное движение заряженных частиц в замкнутом контуре.
  • Условно принято считать, что ЭДС — это работа в 1 Джоуль, совершаемая при перемещении заряда в 1 Кулон и измеряется в Вольтах.
  • ЭДС = 1Джоуль/1Кулон= 1 Вольт.
  • Ну а теперь давайте переключим свое внимание на напряжение.

Что такое напряжение

Итак, напряжение измеряется в аналогичных величинах, то есть в Вольтах. И напряжение — это разница потенциалов между двумя точками цепочки. Причем данные потенциалы рассматриваются только в электростатическом поле.

Получается, если мы с вами будем перемещать заряд величиной в 1 Кулон и точку №1 в точку №2, мы так же будем совершать работу в 1 Джоуль, при том условии, что разница потенциалов между точками будет равна 1 Вольт.

Вроде одно и то же, но в случае с напряжением обязательным условием является наличие электростатического поля. А откуда оно взялось? Так вот источником этого поля и является подключенный к цепи источник ЭДС.

Если провести аналогию с водонапорной башней, то можно представить следующую картинку:

yandex.ru

На картинке наглядно продемонстрирована разница между ЭДС и напряжением. В правой части жидкость перемещается за счет давления (напряжения), а в левой части за счет работы сторонних сил (электродвижущей силы).

Получается, если мы с вами возьмем любой гальванический элемент, например, батарейку и измерим с помощью мультиметра его напряжение без подключенной нагрузки, то таким образом мы получим величину ЭДС.

Если же мы с вами создадим замкнутую цепь, в которую будет включена любая нагрузка, то, измеряя напряжение на тех же выводах батарейки, мы с вами увидим уже напряжение, и оно будет несколько меньше чем величина ЭДС.

Это связано с тем, что внутри любого источника ЭДС присутствует внутреннее сопротивление и когда мы подключаем нагрузку, происходит падение напряжения не только на концах нагрузки, но и на самом внутреннем сопротивлении источника ЭДС.

Если вам понравилась статья, тогда оцените ее лайком и спасибо, что уделили свое внимание.

В чем разница между электрическим потенциалом, разностью потенциалов (pd), напряжением и электродвижущей силой (emf)? — спросисеть

В любом случае, простой ответ: ЭДС не является силой в механическом смысле. Он измеряет объем работы, которую необходимо проделать, чтобы единичный заряд перемещался по замкнутому контуру проводящего материала.

Давайте сделаем это более понятным. В статическом случае (игнорируя изменение во времени любого магнитного поля), электрическое поле в точке может быть получено исключительно из скаляра как отрицательное значение градиента этого скаляра. Этот скаляр в любой точке называется «электрическим потенциалом» в этой точке.

Если две точки имеют разные потенциалы, мы говорим, что существует разность потенциалов. Очевидно, что разница в потенциалах, а не их абсолютные значения. Поэтому можно произвольно присвоить нулевое значение для некоторой фиксированной точки, потенциал которой можно считать постоянной, и сравнить потенциалы других точек по отношению к ней.

Таким образом, не нужно всегда говорить о разности потенциалов, а просто о потенциалах.

  • Теперь, часто этот «электрический потенциал» в некоторой точке в проводнике или диэлектрике называют «напряжением» в этой точке, назначая значение напряжения равным нулю для земли, поскольку потенциал земли постоянен для всех практических целей.
  • Если нет изменений магнитного поля, то работа, выполненная единичным зарядом в замкнутом контуре, будет 0 0 , Но если магнитное поле меняется, то оно будет отличным от нуля. Напомним формулу:
  • ∇ × E = — ∂ В ∂ T , ∇ × Е знак равно — ∂ В ∂ T ,

На самом деле это означает, что электрическое поле, полученное исключительно из скалярного потенциала, не может поддерживать электрический ток в замкнутой цепи. Таким образом, эдс подразумевает наличие какого-то источника, отличного от источника, который может производить только скалярный потенциал.

Следующее уравнение рассказывает всю историю:

Е = — ∇ ϕ — ∂ ∂ T , Е знак равно — ∇ φ — ∂ ∂ T ,

где φ φ скалярный потенциал и это векторный потенциал. Рон Маймон

Люди иногда отрицают не потому, что вы не правы, а потому, что вы повторяете ответы других людей, не добавляя ничего нового.

ganzewoort

sb1, ваше объяснение снова не в состоянии объяснить ЭДС разомкнутой цепи. Что еще интереснее, мне любопытно услышать ваше объяснение того, как закон Фарадея, о котором вы говорите, учитывает падение напряжения, измеренное на однополярном генераторе. Возможно, это отдельный вопрос, который нужно задать в stackexchange.

ganzewoort

Я согласен со скалярным потенциалом (я думаю, вы очень хорошо это объяснили и не повторяете того, что было сказано до сих пор). Однако скалярный потенциал — это всего лишь математическая конструкция, созданная для удобства, которая не присуща явлениям. Я добавляю отдельный вопрос относительно однополярного генератора.

19. Эдс, разность потенциалов и напряжение

Электродвижущая
сила
 (ЭДС) —
скалярная физическая
величина,
характеризующая работу сторонних (н
епотенциальных) сил висточниках постоянного
или переменного тока. В замкнутом
проводящем контуре ЭДС равна работе этих
сил по перемещению единичного
положительного заряда вдоль
контура.

ЭДС
можно выразить через напряжённость
электрического поля сторонних
сил ().
В замкнутом контуре ()
тогда ЭДС будет равна:

,
где —
элемент длины контура.

Причиной
электродвижущей силы может стать
изменение магнитного
поля в
окружающем пространстве. Это явление
называетсяэлектромагнитной
индукцией.
Величина ЭДС индукции в контуре
определяется выражением

где — поток
магнитного поля через
замкнутую поверхность ,
ограниченную контуром. Знак «−» перед
выражением показывает, что индукционный
ток, созданный ЭДС индукции, препятствует
изменению магнитного потока в контуре
(см. правило
Ленца).

Электрическое
напряжение между двумя точками
электрической цепи или электрического
поля, равно работе электрического поля
по перемещению единичного положительного
заряда из одной точки в другую. В
потенциальном электрическом поле эта
работа не зависит от пути, по которому
перемещается заряд; в этом случае Э. н.
между двумя точками совпадает с разностью
потенциалов между ними.

Если поле
непотенциально, то напряжение зависит
от того пути, по которому перемещается
заряд между точками. Непотенциальные
силы, называются сторонними, действуют
внутри любого источника постоянного
тока (генератора, аккумулятора,
гальванического элемента и др.).

Под
напряжением на зажимах источника тока
всегда понимают работу электрического
поля по перемещению единичного
положительного заряда вдоль пути,
лежащего вне источника; в этом случае
Э. н. равно разности потенциалов на
зажимах источника и определяется законом
Ома: U = IR—E, где I — сила тока, R — внутреннее
сопротивление источника, а E — его
электродвижущая сила (эдс).

При разомкнутой
цепи (I = 0) напряжение по модулю равно
эдс источника. Поэтому эдс источника
часто определяют как Э. н. на его зажимах
при разомкнутой цепи.

В случае переменного
тока Э. н. обычно характеризуется
действующим (эффективным) значением,
которое представляет собой среднеквадратичное
за период значение напряжения.

Напряжение
на зажимах источника переменного тока
или катушки индуктивности измеряется
работой электрического поля по перемещению
единичного положительного заряда вдоль
пути, лежащего вне источника или катушки.

Вихревое (непотенциальное) электрическое
поле на этом пути практически отсутствует,
и напряжение равно разности потенциалов.

Электродвижущая
сила (ЭДС) — физическая величина,
характеризующая работу сторонних
(непотенциальных) сил в источниках
постоянного или переменного тока. В
замкнутом проводящем контуре ЭДС равна
работе этих сил по перемещению единичного
положительного заряда вдоль контура.

  • Наименование и
    обозначение производной единицы СИ:
  • международное
    – volt,
    V
  • русское
    – вольт,
    В
  • Выражение через
    основные и производные единицы СИ:
  • 1 V = 1 W / A

В чем основная разница между напряжением, эдс и разностью потенциалов?

В нескольких концепциях электричества идею «напряжения» или «электрического потенциала», вероятно, труднее всего понять. Это также очень сложно объяснить. Это головная боль и для ученика, и для учителя. Чтобы понять напряжение, полезно сначала немного понять его ближайшего родственника — магнетизм.

«электростатическое поле» или «электронное поле». Этот второй тип поля очень похож на магнетизм. Он невидим, у него есть линии потока, и он может притягивать и отталкивать объекты. Однако это не магнетизм, это нечто отдельное. Это напряжение.

Большинство людей знают о магнитных полях, но не об электронных полях или «полях напряжения». Частично это объясняется тем, что магнетизм объясняется в школе, но по некоторым причинам поля напряжения скрыты под названием «статическое электричество». Электронные поля никогда не упоминаются в научных учебниках для начинающих.

Это странно, так как напряжение и «статическое электричество» идут вместе. Всякий раз, когда отрицательный заряд притягивает положительный заряд, между зарядами должны существовать невидимые поля напряжения. Напряжение вызывает притяжение между противоположными зарядами; поля напряжения достигают пространства.

В действительности, «статическое» электричество не имеет ничего общего с движением (или со статичностью). Вместо этого статическое электричество связано с высоким напряжением. Разденьте коврик, и вы зарядите свое тело до нескольких тысяч вольт.

Когда вы снимаете шерстяной носок с сушилки для белья, и все волокна выходят наружу, волокна следуют за невидимыми линиями напряжения в воздухе. Волокна ткани — это «железные опилки», которые делают рисунки напряжения видимыми.

И всякий раз, когда заряды внутри проводника вынуждены течь, они движутся только потому, что их движет поле напряжения, которое проходит по длине провода. Электронные поля вызывают ускорение заряда: напряжение вызывает ток. Напряжение вызывает осушение, но оно также вызывает электрические токи в проводах.

Другой способ сказать это: токи в электрических цепях вызваны «статическим электричеством», а «статическое электричество» не обязательно является статическим. Связь между напряжением и «статическим» электричеством плохо объяснена в книгах, и это одна из главных причин, почему напряжение кажется таким сложным и загадочным.

Простая математика за «напряжением» Чтобы быть более конкретным, «Напряжение» — это способ использования чисел для описания электрического поля. Электрические поля или «электронные поля» измеряются в вольтах на расстоянии; вольт на сантиметр например.

Более сильное электронное поле имеет больше вольт на сантиметр, чем более слабое. Напряжение и электронные поля в основном одно и то же: если электронные поля похожи на склон горы, то вольт похож на различные высоты каждой отдельной точки на горе.

Склон горы может заставить валун начать катиться. То же самое можно сказать о разной высоте разных точек на горе, это просто еще один способ описать одно и то же.

Электронное поле можно рассматривать в терминах сложенных слоев эквипотенциальных поверхностей или в виде совокупностей линий потока. «Напряжение» и «силовые линии» — два способа описать одну и ту же базовую концепцию.

  • Когда у вас есть электронные поля, у вас есть напряжение. Электронные поля могут существовать в воздухе, как и напряжение. Всякий раз, когда у вас есть высокое напряжение на коротком расстоянии, у вас есть сильные электронные поля. Всякий раз, когда электронное поле притягивает или отталкивает объект, мы можем сказать, что объект управляется напряжением в пространстве вокруг объекта. магнетизм — это «то, что включает в себя магнитные поля», тогда что такое «то, что включает в себя электрические поля»?

Вольтаж!

Возьмите несколько гвоздей с помощью магнита, и это пример магнетизма, затем возьмите несколько кусочков бумаги с натертым мехом воздушным шаром, и это пример напряжения. Какие три вида невидимых полей? Гравитация, магнетизм … и напряжение!

Возможно, нам следует изменить слово «Электромагнетизм» на «Напряжение магнетизма»? (Оскал!)

  • просто … Мы определяем напряжение как количество потенциальной энергии между двумя точками в цепи. Одна точка имеет больше заряда, чем другая. Эта разница в заряде между двумя точками называется напряжением и учитывает резервуар для воды на определенной высоте над землей. На дне этого бака есть шланг.
  1. При описании напряжения, тока и сопротивления распространенной аналогией является резервуар для воды. В этой аналогии заряд представлен количеством воды, напряжение представлено давлением воды, а ток представлен потоком воды. Так что для этой аналогии, помните:
  • Water = ChargePressure = VoltageFlow = ТекущийЭлектромоторная сила
  1. Что такое электродвижущая сила? Разность потенциалов между двумя выводами ячейки называется электродвижущей силой в разомкнутой цепи. Электродвижущая сила всегда больше разности потенциалов. Электродвижущая сила не зависит от сопротивления цепи. Электродвижущая сила создает разность потенциалов во всей цепи.

Электродвижущая сила

У этого термина существуют и другие значения, см. Сила (значения).

Классическая электродинамика
Электричество · Магнетизм
Электростатика
Закон Кулона Теорема Гаусса Электрический дипольный момент Электрический заряд Электрическая индукция Электрическое поле Электростатический потенциал
Магнитостатика
Закон Био — Савара — Лапласа Закон Ампера Магнитный момент Магнитное поле Магнитный поток Магнитная индукция
Электродинамика
Векторный потенциал Диполь Потенциалы Лиенара — Вихерта Сила Лоренца Ток смещения Униполярная индукция Уравнения Максвелла Электрический ток Электродвижущая сила Электромагнитная индукция Электромагнитное излучение Электромагнитное поле
Электрическая цепь
Закон Ома Законы Кирхгофа Индуктивность Радиоволновод Резонатор Электрическая ёмкость Электрическая проводимость Электрическое сопротивление Электрический импеданс
Ковариантная формулировка
Тензор электромагнитного поля Тензор энергии-импульса 4-потенциал 4-ток
Известные учёные
Генри Кавендиш Майкл Фарадей Никола Тесла Андре-Мари Ампер Густав Роберт Кирхгоф Джеймс Клерк Максвелл Оливер Хевисайд Генрих Рудольф Герц Альберт Абрахам Майкельсон Роберт Эндрюс Милликен
См. также: Портал:Физика

Электродвижущая сила (ЭДС) — скалярная физическая величина, характеризующая работу сторонних сил (то есть любых сил, кроме электростатических и диссипативных) действующих в квазистационарных цепях постоянного или переменного тока. В замкнутом проводящем контуре ЭДС равна работе этих сил по перемещению единичного положительного заряда вдоль всего контура[1][2].

По аналогии с напряжённостью электрического поля вводят понятие напряжённость сторонних сил

E

e
x

{displaystyle {vec {E}}_{ex}}

, под которой понимают векторную физическую величину, равную отношению сторонней силы, действующей на пробный электрический заряд к величине этого заряда. Тогда в замкнутом контуре

L

{displaystyle L}

ЭДС будет равна:

E

=

L

E

e
x

d
l

,

{displaystyle {mathcal {E}}=oint limits _{L}{vec {E}}_{ex}cdot {vec {dl}},}

где

d
l

{displaystyle {vec {dl}}}

 — элемент контура.

ЭДС так же, как и напряжение, в Международной системе единиц (СИ) измеряется в вольтах.
Можно говорить об электродвижущей силе на любом участке цепи. Это удельная работа сторонних сил не во всем контуре, а только на данном участке.

ЭДС гальванического элемента есть работа сторонних сил при перемещении единичного положительного заряда внутри элемента от одного полюса к другому. Работа сторонних сил не может быть выражена через разность потенциалов, так как сторонние силы непотенциальны и их работа зависит от формы траектории.

Так, например, работа сторонних сил при перемещении заряда между клеммами источника тока вне самого́ источника равна нулю.

Эдс и закон ома

Электродвижущая сила источника связана с электрическим током, протекающим в цепи, соотношениями закона Ома. Закон Ома для неоднородного участка цепи имеет вид[1]:

φ

1

φ

2

+

E

=
I
R
,

{displaystyle varphi _{1}-varphi _{2}+{mathcal {E}}=IR,}

где

φ

1

φ

2

{displaystyle varphi _{1}-varphi _{2}}

— разность между значениями потенциала в начале и в конце участка цепи,

I

{displaystyle I}

— сила тока, текущего по участку, а

R

{displaystyle R}

— сопротивление участка.

Если точки 1 и 2 совпадают (цепь замкнута), то

φ

1

φ

2

=
0

{displaystyle varphi _{1}-varphi _{2}=0}

и предыдущая формула переходит в формулу закона Ома для замкнутой цепи[1]:

E

=
I
R
,

{displaystyle {mathcal {E}}=IR,}

где теперь

R

{displaystyle R}

полное сопротивление всей цепи.

В общем случае полное сопротивление цепи складывается из сопротивления внешнего по отношению к источнику тока участка цепи (

R

e

{displaystyle R_{e}}

) и внутреннего сопротивления самого́ источника тока (

r

{displaystyle r}

). С учётом этого следует:

E

=
I

R

e

+
I
r
.

{displaystyle {mathcal {E}}=IR_{e}+Ir.}

Эдс источника тока

Если на участке цепи не действуют сторонние силы (однородный участок цепи) и, значит, источника тока на нём нет, то, как это следует из закона Ома для неоднородного участка цепи, выполняется:

φ

1

φ

2

=
I
R
.

{displaystyle varphi _{1}-varphi _{2}=IR.}

Значит, если в качестве точки 1 выбрать анод источника, а в качестве точки 2 — его катод, то для разности между потенциалами анода

φ

a

{displaystyle varphi _{a}}

и катода

φ

k

{displaystyle varphi _{k}}

можно записать:

φ

a

φ

k

=
I

R

e

,

{displaystyle varphi _{a}-varphi _{k}=IR_{e},}

где как и ранее

R

e

{displaystyle R_{e}}

— сопротивление внешнего участка цепи.

Из этого соотношения и закона Ома для замкнутой цепи, записанного в виде

E

=
I

R

e

+
I
r

{displaystyle {mathcal {E}}=IR_{e}+Ir}

нетрудно получить

φ

a

φ

k

E

=

R

e

R

e

+
r

{displaystyle {frac {varphi _{a}-varphi _{k}}{mathcal {E}}}={frac {R_{e}}{R_{e}+r}}}

и затем

φ

a

φ

k

=

R

e

R

e

+
r

E

.

{displaystyle varphi _{a}-varphi _{k}={frac {R_{e}}{R_{e}+r}}{mathcal {E}}.}

Из полученного соотношения следуют два вывода:

  1. Во всех случаях, когда по цепи течёт ток, разность потенциалов между клеммами источника тока

    φ

    a

    φ

    k

    {displaystyle varphi _{a}-varphi _{k}}

    меньше, чем ЭДС источника.

  2. В предельном случае, когда

    R

    e

    {displaystyle R_{e}}

    бесконечно (цепь разорвана), выполняется

    E

    =

    φ

    a

    φ

    k

    .

    {displaystyle {mathcal {E}}=varphi _{a}-varphi _{k}.}

Таким образом, Эдс источника тока равна разности потенциалов между его клеммами в состоянии, когда источник отключён от цепи[1].

Эдс индукции

Причиной возникновения электродвижущей силы в замкнутом контуре может стать изменение потока магнитного поля, пронизывающего поверхность, ограниченную данным контуром. Это явление называется электромагнитной индукцией. Величина Эдс индукции в контуре определяется выражением

E

=

d
Φ

d
t

,

{displaystyle {mathcal {E}}=-{frac {dPhi }{dt}},}

где

Φ

{displaystyle Phi }

— поток магнитного поля через замкнутую поверхность, ограниченную контуром.

Знак «−» перед выражением показывает, что индукционный ток, созданный Эдс индукции, препятствует изменению магнитного потока в контуре (см. правило Ленца).

В свою очередь причиной изменения магнитного потока может быть как изменение магнитного поля, так и движение контура в целом или его отдельных частей.

Неэлектростатический характер ЭДС

Внутри источника ЭДС ток течёт в направлении, противоположном нормальному.

Это невозможно без дополнительной силы неэлектростатической природы, преодолевающей силу электрического отталкивания

Как показано на рисунке, электрический ток, нормальное направление которого — от «плюса» к «минусу», внутри источника ЭДС (например, внутри гальванического элемента) течёт в противоположном направлении.

Направление от «плюса» к «минусу» совпадает с направлением электростатической силы, действующей на положительные заряды.

Поэтому для того, чтобы заставить ток течь в противоположном направлении, необходима дополнительная сила неэлектростатической природы (центробежная сила, сила Лоренца, силы химической природы, сила со стороны вихревого электрического поля) которая бы преодолевала силу со стороны электростатического поля. Диссипативные силы, хотя и противодействуют электростатическому полю, не могут заставить ток течь в противоположном направлении, поэтому они не входят в состав сторонних сил, работа которых используется в определении ЭДС.

Сторонние силы

Сторонними силами называются силы, вызывающие перемещение электрических зарядов внутри источника постоянного тока против направления действия сил электростатического поля.

Например, в гальваническом элементе или аккумуляторе сторонние силы возникают в результате электрохимических процессов, происходящих на границе соприкосновения электрода с электролитом; в электрическом генераторе постоянного тока сторонней силой является сила Лоренца[3].

См. также

  • Правила Кирхгофа

Примечания

  1. 1 2 3 4 Сивухин Д. В. Общий курс физики. — М.: Физматлит, МФТИ, 2004. — Т. III. Электричество. — С. 193—194. — 656 с. — ISBN 5-9221-0227-3.
  2. Калашников С. Г. Общий курс физики. — М.: Гостехтеориздат, 1956. — Т. II. Электричество. — С. 146, 153. — 664 с.
  3. Кабардин О. Ф. Физика. — М., Просвещение, 1985. — Тираж 754 000 экз. — с. 131
В этой статье не хватает ссылок на источники информации.Информация должна быть проверяема, иначе она может быть поставлена под сомнение и удалена.Вы можете отредактировать эту статью, добавив ссылки на авторитетные источники.Эта отметка установлена 19 июня 2018 года.

Электродвижущая сила (ЭДС) — определение, расшифровка, единица измерения, схемы и формулы

В статье понятным и простым языком объясняется явление ЭДС. Представлена краткая историческая справка, рассмотрены типы ЭДС и её связь с параметрами электрической цепи. Текст подкреплён элементарными формулами.

Что такое ЭДС в физике – физический смысл

Электрический ток будет проходить через проводник только в том случае, если единовременно соблюдаются два простых условия:

  1. В проводнике присутствуют свободные электроны (например, в металлах электронов, не связанных с атомом, большинство).
  2. В проводнике присутствует сила, вынуждающая электроны двигаться.

Допустим, на концы электрода подали разные по знаку заряды, которые под действием кулоновской силы начинают притягиваться друг к другу.

Однако без сторонних сил электрическое поле, появившееся в результате такого взаимодействия, исчезнет, как только электроны придут в равновесие, поэтому для поддержания в проводнике электрического тока нужен источник питания, например батарейка.

ВАЖНО: электроны могут перемещаться только силами неэлектрического происхождения (сторонними силами), ярким примером которых являются химические процессы, происходящие в батарее.

При замыкании цепи «проводник – источник тока» электроны вновь начнут движение друг к другу, но как только положительный заряд приблизится к отрицательному, сторонние силы перенесут его обратно.

Так, работа этих сторонних сил по переносу единичного положительного заряда называется ЭДС.

Что такое ЭДС в электротехнике?

В электротехнике ЭДС характеризует источники питания и создаёт и поддерживает в течение длительного периода времени разность потенциалов. Численно ЭДС равна работе, которую должны совершить либо сторонние силы, чтобы переместить положительный заряд внутри источника, либо сам источник, чтобы провести заряд по цепи. Таким образом, формула для вычисления ЭДС имеет вид:

E = A / q,

где E – ЭДС,

А – работа,

q – заряд.

ЭДС необходима для поддержания в цепи постоянного тока, причём в технике применяется несколько видов ЭДС.

ВидОбласть применения
ХимическаяБатарейки и аккумуляторы
ТермоэлектрическаяХолодильники и термопары
ИндукционнаяЭлектродвигатели, генераторы и трансформаторы
ФотоэлектрическаяФотоэлементы
ПьезоэлектрическаяПьезоэлементы, датчики, кварцевые генераторы

СПРАВКА: в теории существует идеальный источник ЭДС – генератор с нулевым внутренним сопротивлением, мощность которого приравнивается к бесконечности.

Расшифровка ЭДС

Аббревиатура ЭДС общепринятая и расшифровывается как «электродвижущая сила».

СПРАВКА: понятие ЭДС введено Георгом Омом в 1827 году, а её значение определено Густавом Кирхгофом в 1857.

В чём измеряется ЭДС – единица измерения

Уже было отмечено, что ЭДС – отношение работы к заряду, то есть:

Единица измерения E = 1 джоуль (Дж) / 1 кулон (Кл) = 1 вольт (В).

Таким образом, ЭДС, как напряжение, измеряется в вольтах. Причём на практике часто используют более крупные и мелкие единицы:

  • киловольт (кВ): 1 кВ = 103 В;
  • милливольт (мВ): 1мВ = 10-3 В;
  • микровольт (мкВ): 1 мкВ = 10-6 В.

Чем отличается ЭДС от напряжения?

Известно, что напряжение характеризует работу электрического поля по переносу положительного заряда и измеряется в вольтах. Таким образом, на первый взгляд ЭДС и напряжение мало чем отличаются друг от друга, однако различие между этими понятиями есть и весьма существенное.

Схема с ЭДС

В реальной электрической цепи присутствует внутреннее сопротивление, на котором происходит падение напряжения. Причём, если разомкнуть цепь и соединить вольтметр с батареей, он покажет значение ЭДС – 1,5 В, но при подключении нагрузки, например лампочки, на клеммах будет меньшее значение. Эти процессы описываются законом Ома для полной цепи.

То есть основная разница между величинами состоит в том, что напряжение зависит от нагрузки и тока в цепи, а ЭДС – от источника питания.

СПРАВКА: в идеальной электрической цепи, где отсутствует внутреннее сопротивление, между напряжением и ЭДС не будет разницы.

ЭДС электромагнитной индукции

29 августа 1831 года Майкл Фарадей открыл электромагнитную индукцию – явление возникновения электрического тока при движении замкнутого проводящего контура в магнитном поле или при изменении в течение времени этого поля.

Фарадей в ходе эксперимента обнаружил, что возникающая ЭДС зависит от скорости изменения магнитного потока через поверхность замкнутого контура, но не зависит от причины этого изменения.

Eинд = — dФ / dt,

где Eинд – ЭДС индукции,

Ф – магнитный поток, измеряемый в веберах (Вб),

t – время.

Знак дифференциала d характеризует изменение величин, а минус перед отношением отражает правило Ленца, согласно которому индукционный ток, вызванный ЭДС индукции, направлен таким образом, чтобы противодействовать изменению магнитного потока.

ЭДС источника тока

Электродвижущая сила источника тока характеризует его способность создавать и поддерживать разность потенциалов на зажимах.

ВНИМАНИЕ: ЭДС может возникнуть в источнике и при разомкнутой цепи, при этом данную ситуацию называют «холостым ходом», а величина силы приравнивается к разнице потенциалов.

ЭДС индукции в движущихся проводниках

Пусть в однородном магнитном поле с постоянной скоростью движется проводник. Тогда на каждый свободный электрон проводника будет действовать сила Лоренца, под действием которой отрицательные частицы начнут движение. В результате один из концов проводника зарядится отрицательно, второй – положительно, то есть возникнет разница потенциалов. Исходя из этого можно сделать вывод, что данный проводник в такой ситуации будет представлять собой источник тока, а разность потенциалов на его концах, по сути, представляет собой ЭДС.

Eинд = Blvsinα,

где B – вектор индукции магнитного поля,

l – длина проводника,

v – скорость его перемещения в магнитном поле,

αугол направления движения к направлению действия поля, то есть угол между B и v.

ЭДС катушки индуктивности

Особенность катушки – способность создавать магнитное поле, если по её проводу течёт электрический ток, что называется индуктивностью.

Схема ЭДС с катушкой индуктивности

Допустим, собрана схема с катушкой с железным сердечником и лампочкой, подключенной параллельно. Если сначала замкнуть цепь, дав току, протекающему в неё, установиться, а потом резко разомкнуть, лампочка резко вспыхнет. Что свидетельствует о том, что при отключении цепи от источника питания ток из катушки перешёл в лампу. То есть ток в катушке был и имел вокруг себя магнитное поле, после исчезновения которого возникла ЭДС.

Такая электродвижущая сила называется ЭДС самоиндукции, так как она появилась от собственного магнитного поля катушки.

ЭДС гальванического элемента

Гальванический элемент – это источник тока, создающий его из химической энергии. Рассмотрим элемент Даниэля-Якоби, представляющий собой цинковую и медную пластины в соответствующих растворах сульфатов, соединённые между собой электролитом. Если соединить пластины металлическим стержнем, начнётся перераспределение зарядов: свободные электроны будут перемещаться к электроду с менее отрицательным зарядом (медной пластине). То есть возникнет электрический ток. Его работа будет максимальной в том случае, когда процессы на электродах (окисление и восстановление вследствие изменения числа электронов) будут протекать бесконечно медленно.

ЭДС гальванического элемента – максимальная разность потенциалов, возможная в такой ситуации.

Мощность через ЭДС

Известно, что мощность тока – это работа, совершаемая в единицу времени, то есть:

P = A / Δt,

где P – мощность.

Кроме этого, существует формула для вычисления мощности на участке цепи, связывающая эту величину с напряжением и током:

P = UI,

где U – напряжение,

I – ток.

В случае, если участок цепи содержит источник тока, имеющий ЭДС, формула будет иметь вид:

P = (u1u2)∙I + EI,

где u1u2 – разность потенциалов.

ЭДС через магнитный поток

Было отмечено, что Фарадей установил соотношение зависимости ЭДС от магнитного потока:

E = — ΔФ / Δt.

Известно, что магнитный поток можно найти, опираясь на выражение:

Ф = BScosα,

где S – площадь поверхности, через которую проходит поток,

α – угол между вектором магнитной индукции и нормалью к поверхности.

Для некоторого упрощения допустим, что плоскость контура располагается перпендикулярно к магнитному полю, то есть α = 0. Учитывая, что ΔФ = Ф2 — Ф1 = B(S2S1), формула ЭДС может иметь вид:

E = — B(S2S1) / Δt.

Напряжение через ЭДС

Согласно закону Ома для участка цепи:

I = U / R,

где R – сопротивление.

Этот же закон для полной цепи имеет вид:

I = E / (R+r),

где r – сопротивление источника питания.

Пусть количество электронов, произведённых источником тока, равно количеству зарядов, которые «ушли» в цепь. Тогда справедливо равенство:

U / R = E / (R+r).

Путём элементарных математических действий можно получить связь напряжения и ЭДС:

U = ER / (R+r).

СПРАВКА: для идеальной цепи: U = E.

Как обозначается ЭДС на схеме?

Источник ЭДС обычно изображается буквой «Е», расположенной рядом со стрелкой, помещённой в круг. Рассмотрим несколько схем, встречающихся на практике.

Как обозначается ЭДС на схеме

На рисунке под буквой «а» изображён идеальный источник ЭДС, под «б» – реальный источник, обладающий внутренним сопротивлением, под «в» – элементарная электрическая цепь: реальный источник ЭДС и потребитель.

Как можно повысить точность измерения ЭДС источника тока?

Одним из способов повышения точности является проведение серии измерений, что позволит снизить риск случайных ошибок. Кроме этого, в серию испытания можно включить измерение разности потенциалов, тока, внутреннего сопротивления источника, а после вычислить среднее значение требуемой величины.

Наиболее простой способ повышение точности – использование вольтметра высокого класса точности.

Закон Ома — онлайн калькулятор

Чтобы посчитать Закон Ома воспользуйтесь нашим очень удобным онлайн калькулятором:

Закон Ома для участка цепи

Закон Ома для участка цепи гласит, что сила тока (I) на участке электрической цепи прямо пропорциональна напряжению (U) на концах участка цепи и обратно пропорциональна его сопротивлению (R).

Онлайн калькулятор

Найти силу тока

Формула

I = U/R

Пример

Если напряжение на концах участка цепи U = 12 В, а его электрическое сопротивление R = 2 Ом, то:

Сила тока на этом участке I = 12/2= 6 А

Найти напряжение

Формула

U = I ⋅ R

Пример

Если сила тока на участке цепи I = 6 А, а электрическое сопротивление этого участка R = 2 Ом, то:

Напряжение на этом участке U = 6⋅2 = 12 В

Найти сопротивление

Формула

R = U/I

Пример

Если напряжение на концах участка цепи U = 12 В, а сила тока на участке цепи I = 6 А, то:

Электрическое сопротивление на этом участке R = 12/6 = 2 Ом

Закон Ома для полной цепи

Закон Ома для полной цепи гласит, что сила тока в цепи пропорциональна действующей в цепи электродвижущей силе (ЭДС) и обратно пропорциональна сумме сопротивлений цепи и внутреннего сопротивления источника.

Онлайн калькулятор

Найти силу тока

Формула

I = ε/R+r

Пример

Если ЭДС источника напряжения ε = 12 В, сопротивление всех внешних элементов цепи R = 4 Ом, а внутреннее сопротивление источника напряжения r = 2 Ом, то:

Сила тока I = 12/4+2 = 2 А

Найти ЭДС

Формула

ε = I ⋅ (R+r)

Пример

Если сила тока в цепи I = 2A, сопротивление всех внешних элементов цепи R = 4 Ом, а внутреннее сопротивление источника напряжения r = 2 Ом, то:

ЭДС ε = 2 ⋅ (4+2) = 12 В

Найти внутреннее сопротивление источника напряжения

Формула

r = ε/I— R

Пример

Если сила тока в цепи I = 2A, сопротивление всех внешних элементов цепи R = 4 Ом, а ЭДС источника напряжения ε = 12 В, то:

Внутреннее сопротивление источника напряжения r = 12/2 — 4 = 2 Ом

Найти сопротивление всех внешних элементов цепи

Формула

R = ε/I— r

Пример

Если сила тока в цепи I = 2A, внутреннее сопротивление источника напряжения r = 2 Ом, а ЭДС источника напряжения ε = 12 В, то:

Сопротивление всех внешних элементов цепи: R = 12/2 — 2 = 4 Ом

См. также

10.2: Электродвижущая сила — Physics LibreTexts

Цели обучения

К концу раздела вы сможете:

  • Опишите электродвижущую силу (ЭДС) и внутреннее сопротивление батареи
  • Объясните основную работу аккумулятора

Если вы забыли выключить автомобильные фары, они постепенно тускнеют по мере разрядки аккумулятора. Почему они не мигают внезапно, когда батарея разряжена? Их постепенное затемнение означает, что выходное напряжение батареи уменьшается по мере разряда батареи.Причина снижения выходного напряжения для разряженных батарей заключается в том, что все источники напряжения состоят из двух основных частей — источника электрической энергии и внутреннего сопротивления. В этом разделе мы исследуем источник энергии и внутреннее сопротивление.

Введение в электродвижущую силу

Voltage имеет множество источников, некоторые из которых показаны на рисунке \ (\ PageIndex {2} \). Все такие устройства создают разность потенциалов и могут подавать ток, если подключены к цепи.Особый тип разности потенциалов известен как электродвижущая сила (ЭДС) . ЭДС — это вовсе не сила, но термин «электродвижущая сила» используется по историческим причинам. Он был изобретен Алессандро Вольта в 1800-х годах, когда он изобрел первую батарею, также известную как гальваническая батарея . Поскольку электродвижущая сила не является силой, принято называть эти источники просто источниками ЭДС (произносимыми буквами «ee-em-eff»), а не источниками электродвижущей силы.

Рисунок \ (\ PageIndex {1} \): различные источники напряжения. а) ветряная электростанция Бразос в Флуванна, штат Техас; (б) Красноярская плотина в России; (c) солнечная ферма; (d) группа никель-металлогидридных батарей. Выходное напряжение каждого устройства зависит от его конструкции и нагрузки. Выходное напряжение равно ЭДС только при отсутствии нагрузки. (кредит a: модификация работы «Leaflet» / Wikimedia Commons; кредит b: модификация работы Алекса Полежаева; кредит c: модификация работы Министерства энергетики США; кредит d: модификация работы Тиаа Монто)

Если Электродвижущая сила — это вообще не сила, тогда что такое ЭДС и что является источником ЭДС? Чтобы ответить на эти вопросы, рассмотрим простую схему лампы 12 В, подключенной к батарее 12 В, как показано на рисунке \ (\ PageIndex {2} \).Батарея , может быть смоделирована как устройство с двумя выводами, которое поддерживает один вывод с более высоким электрическим потенциалом, чем второй вывод. Более высокий электрический потенциал иногда называют положительной клеммой и обозначают знаком плюс. Клемму с более низким потенциалом иногда называют отрицательной клеммой и обозначают знаком минус. Это источник ЭДС.

Рисунок \ (\ PageIndex {2} \): Источник ЭДС поддерживает на одном выводе более высокий электрический потенциал, чем на другом выводе, действуя как источник тока в цепи.

Когда источник ЭДС не подключен к лампе, нет чистого потока заряда внутри источника ЭДС. Как только батарея подключена к лампе, заряды перетекают от одной клеммы батареи через лампу (в результате чего лампа загорается) и обратно к другой клемме батареи. Если мы рассмотрим протекание положительного (обычного) тока, положительные заряды покидают положительный вывод, проходят через лампу и попадают в отрицательный вывод.

Положительный поток тока полезен для большей части анализа схем в этой главе, но в металлических проводах и резисторах наибольший вклад в ток вносят электроны, протекающие в направлении, противоположном положительному потоку тока.Поэтому более реалистично рассматривать движение электронов для анализа схемы на рисунке \ (\ PageIndex {2} \). Электроны покидают отрицательную клемму, проходят через лампу и возвращаются к положительной клемме. Чтобы источник ЭДС поддерживал разность потенциалов между двумя выводами, отрицательные заряды (электроны) должны перемещаться с положительного вывода на отрицательный. Источник ЭДС действует как накачка заряда, перемещая отрицательные заряды от положительного вывода к отрицательному для поддержания разности потенциалов.Это увеличивает потенциальную энергию зарядов и, следовательно, электрический потенциал зарядов.

Сила, действующая на отрицательный заряд электрического поля, действует в направлении, противоположном электрическому полю, как показано на рисунке \ (\ PageIndex {2} \). Чтобы отрицательные заряды переместились на отрицательный вывод, необходимо провести работу с отрицательными зарядами. Для этого требуется энергия, которая возникает в результате химических реакций в батарее. Потенциал поддерживается высоким на положительной клемме и низким на отрицательной клемме, чтобы поддерживать разность потенциалов между двумя клеммами.ЭДС равна работе, выполняемой над зарядом на единицу заряда \ (\ left (\ epsilon = \ frac {dW} {dq} \ right) \) при отсутствии тока. Поскольку единицей работы является джоуль, а единицей заряда — кулон, единицей измерения ЭДС является вольт \ ((1 \, V = 1 \, J / C) \).

Напряжение на клеммах \ (V_ {клемма} \) батареи — это напряжение, измеренное на клеммах батареи, когда к клемме не подключена нагрузка. Идеальная батарея — это источник ЭДС, который поддерживает постоянное напряжение на клеммах, независимо от тока между двумя клеммами.Идеальная батарея не имеет внутреннего сопротивления, а напряжение на клеммах равно ЭДС батареи. В следующем разделе мы покажем, что у реальной батареи есть внутреннее сопротивление, а напряжение на клеммах всегда меньше, чем ЭДС батареи.

Происхождение потенциала батареи

ЭДС батареи определяется сочетанием химических веществ и составом выводов батареи. Свинцово-кислотный аккумулятор , используемый в автомобилях и других транспортных средствах, является одним из наиболее распространенных сочетаний химических веществ.На рисунке \ (\ PageIndex {3} \) показана одна ячейка (одна из шести) этой батареи. Катодная (положительная) клемма ячейки соединена с пластиной из оксида свинца, а анодная (отрицательная) клемма подключена к свинцовой пластине. Обе пластины погружены в серную кислоту, электролит для системы.

Рисунок \ (\ PageIndex {3} \): Химические реакции в свинцово-кислотном элементе разделяют заряд, отправляя отрицательный заряд на анод, который соединен со свинцовыми пластинами. Пластины из оксида свинца подключаются к положительному или катодному выводу ячейки.Серная кислота проводит заряд, а также участвует в химической реакции.

Небольшое знание того, как взаимодействуют химические вещества в свинцово-кислотной батарее, помогает понять потенциал, создаваемый батареей. На рисунке \ (\ PageIndex {4} \) показан результат одной химической реакции. Два электрона помещаются на анод , что делает его отрицательным, при условии, что катод подает два электрона. Это оставляет катод положительно заряженным, потому что он потерял два электрона.Короче говоря, разделение заряда было вызвано химической реакцией.

Обратите внимание, что реакция не происходит, если нет замкнутой цепи, позволяющей подавать два электрона на катод. Во многих случаях эти электроны выходят из анода, проходят через сопротивление и возвращаются на катод. Отметим также, что, поскольку в химических реакциях участвуют вещества, обладающие сопротивлением, невозможно создать ЭДС без внутреннего сопротивления.

Рисунок \ (\ PageIndex {4} \): В свинцово-кислотной батарее два электрона прижимаются к аноду элемента, а два электрона удаляются с катода элемента.В результате химической реакции в свинцово-кислотной батарее два электрона помещаются на анод и два электрона удаляются с катода. Для продолжения требуется замкнутая цепь, так как два электрона должны быть доставлены на катод.

Внутреннее сопротивление и напряжение на клеммах

Величина сопротивления прохождению тока внутри источника напряжения называется внутренним сопротивлением . Внутреннее сопротивление батареи r может вести себя сложным образом. Обычно она увеличивается по мере разряда батареи из-за окисления пластин или снижения кислотности электролита.Однако внутреннее сопротивление также может зависеть от величины и направления тока через источник напряжения, его температуры и даже его предыстории. Например, внутреннее сопротивление перезаряжаемых никель-кадмиевых элементов зависит от того, сколько раз и насколько глубоко они были разряжены. Простая модель батареи состоит из идеализированного источника ЭДС \ (\ epsilon \) и внутреннего сопротивления r (рисунок \ (\ PageIndex {5} \)).

Рисунок \ (\ PageIndex {5} \): Батарею можно смоделировать как идеализированную ЭДС \ ((\ epsilon) \) с внутренним сопротивлением ( r ).Напряжение на клеммах аккумулятора равно \ (V_ {terminal} = \ epsilon — Ir \).

Предположим, что внешний резистор, известный как сопротивление нагрузки R , подключен к источнику напряжения, например батарее, как показано на рисунке \ (\ PageIndex {6} \). На рисунке показана модель аккумулятора с ЭДС ε, внутренним сопротивлением r и нагрузочным резистором R , подключенным к его клеммам. При обычном протекании тока положительные заряды покидают положительную клемму батареи, проходят через резистор и возвращаются к отрицательной клемме батареи.Напряжение на клеммах аккумулятора зависит от ЭДС, внутреннего сопротивления и силы тока и равно

.

Примечание

\ [V_ {терминал} = \ epsilon — Ir \]

При заданной ЭДС и внутреннем сопротивлении напряжение на клеммах уменьшается по мере увеличения тока из-за падения потенциала Ir внутреннего сопротивления.

Рисунок \ (\ PageIndex {6} \): Схема источника напряжения и его нагрузочного резистора R . Поскольку внутреннее сопротивление r последовательно с нагрузкой, оно может существенно повлиять на напряжение на клеммах и ток, подаваемый на нагрузку.

График разности потенциалов на каждом элементе цепи показан на рисунке \ (\ PageIndex {7} \). Через цепь проходит ток I , а падение потенциала на внутреннем резисторе равно Ir . Напряжение на клеммах равно \ (\ epsilon — Ir \), что равно падению потенциала на нагрузочном резисторе \ (IR = \ epsilon — Ir \). Как и в случае с потенциальной энергией, важно изменение напряжения. Когда используется термин «напряжение», мы предполагаем, что на самом деле это изменение потенциала, или \ (\ Delta V \).Однако \ (\ Delta \) часто для удобства опускается.

Рисунок \ (\ PageIndex {7} \): график напряжения в цепи батареи и сопротивления нагрузки. Электрический потенциал увеличивает ЭДС батареи из-за химических реакций, выполняющих работу с зарядами. В аккумуляторе происходит снижение электрического потенциала из-за внутреннего сопротивления. Потенциал уменьшается из-за внутреннего сопротивления \ (- Ir \), в результате чего напряжение на клеммах батареи равно \ ((\ epsilon — Ir) \).Затем напряжение уменьшается на ( IR ). Ток равен \ (I = \ frac {\ epsilon} {r + R} \).

Ток через нагрузочный резистор равен \ (I = \ frac {\ epsilon} {r + R} \). Из этого выражения видно, что чем меньше внутреннее сопротивление r , тем больший ток подает источник напряжения на свою нагрузку R . По мере разряда батарей r увеличивается. Если r становится значительной частью сопротивления нагрузки, то ток значительно снижается, как показано в следующем примере.

Пример \ (\ PageIndex {1} \): анализ цепи с батареей и нагрузкой

Данная батарея имеет ЭДС 12,00 В и внутреннее сопротивление \ (0,100 \, \ Омега \). (a) Рассчитайте напряжение на его клеммах при подключении к нагрузке с \ (10.00 \, \ Omega \). (b) Какое напряжение на клеммах при подключении к нагрузке \ (0.500 \, \ Omega \)? (c) Какая мощность рассеивается при нагрузке \ (0.500 \, \ Omega \)? (d) Если внутреннее сопротивление увеличивается до \ (0.500 \, \ Omega \), найдите ток, напряжение на клеммах и мощность, рассеиваемую элементом \ (0.500 \, \ Omega \) загрузка.

Стратегия

Приведенный выше анализ дал выражение для тока с учетом внутреннего сопротивления. Как только ток будет найден, напряжение на клеммах можно рассчитать с помощью уравнения \ (V_ {terminal} = \ epsilon — Ir \). Как только ток будет найден, мы также сможем найти мощность, рассеиваемую резистором.

Решение

  1. Ввод заданных значений ЭДС, сопротивления нагрузки и внутреннего сопротивления в выражение выше дает \ [I = \ frac {\ epsilon} {R + r} = \ frac {12.00 \, V} {10.10 \, \ Omega} = 1.188 \, A. \] Введите известные значения в уравнение \ (V_ {terminal} = \ epsilon — Ir \), чтобы получить напряжение на клеммах: \ [V_ { клемма} = \ epsilon — Ir = 12.00 \, V — (1.188 \, A) (0.100 \, \ Omega) = 11.90 \, V. \] Напряжение на клеммах здесь лишь немного ниже, чем ЭДС, что означает, что ток втягивается этой легкой нагрузкой незначительно.
  2. Аналогично, при \ (R_ {load} = 0.500 \, \ Omega \) ток равен \ [I = \ frac {\ epsilon} {R + r} = \ frac {12.00 \, V} {0.2} {R} \) или \ (IV \), где В, — напряжение на клеммах (в данном случае 10,0 В).
  3. Здесь внутреннее сопротивление увеличилось, возможно, из-за разряда батареи, до точки, где оно равно сопротивлению нагрузки. Как и раньше, мы сначала находим ток, вводя известные значения в выражение, получая \ [I = \ frac {\ epsilon} {R + r} = \ frac {12.00 \, V} {1.00 \, \ Omega} = 12.00 \, A. \] Теперь напряжение на клеммах равно \ [V_ {terminal} = \ epsilon — Ir = 12.00 \, V — (12.2 (0.500 \, \ Omega) = 72.00 \, W. \] Мы видим, что повышенное внутреннее сопротивление значительно снизило напряжение на клеммах, ток и мощность, подаваемую на нагрузку.

Значение

Внутреннее сопротивление батареи может увеличиваться по многим причинам. Например, внутреннее сопротивление перезаряжаемой батареи увеличивается с увеличением количества раз, когда батарея перезаряжается. Повышенное внутреннее сопротивление может иметь двоякое влияние на аккумулятор.Сначала снизится напряжение на клеммах. Во-вторых, аккумулятор может перегреться из-за повышенной мощности, рассеиваемой внутренним сопротивлением.

Упражнение \ (\ PageIndex {1} \)

Если вы поместите провод прямо между двумя выводами батареи, эффективно закоротив клеммы, батарея начнет нагреваться. Как вы думаете, почему это происходит?

Решение

Если к клеммам подключен провод, сопротивление нагрузки близко к нулю или, по крайней мере, значительно меньше внутреннего сопротивления батареи.2р) \). Мощность рассеивается в виде тепла.

Тестеры аккумуляторов

Тестеры батарей, такие как те, что показаны на рисунке \ (\ PageIndex {8} \), используют малые нагрузочные резисторы, чтобы намеренно потреблять ток, чтобы определить, падает ли потенциал клемм ниже допустимого уровня. Хотя измерить внутреннее сопротивление батареи сложно, тестеры батареи могут обеспечить измерение внутреннего сопротивления батареи. Если внутреннее сопротивление высокое, батарея разряжена, о чем свидетельствует низкое напряжение на клеммах.

Рисунок \ (\ PageIndex {8} \): Тестеры батарей измеряют напряжение на клеммах под нагрузкой, чтобы определить состояние батареи. (a) Техник-электронщик ВМС США использует тестер аккумуляторов для проверки больших аккумуляторов на борту авианосца USS Nimitz . Тестер батарей, который она использует, имеет небольшое сопротивление, которое может рассеивать большое количество энергии. (b) Показанное небольшое устройство используется на небольших батареях и имеет цифровой дисплей для индикации допустимого напряжения на клеммах. (кредит А: модификация работы Джейсона А.Джонстон; кредит b: модификация работы Кейта Уильямсона)

Некоторые батареи можно заряжать, пропуская через них ток в направлении, противоположном току, который они подают в прибор. Это обычно делается в автомобилях и батареях для небольших электроприборов и электронных устройств (Рисунок \ (\ PageIndex {9} \)). Выходное напряжение зарядного устройства должно быть больше, чем ЭДС аккумулятора, чтобы ток через него реверсировал. Это приводит к тому, что напряжение на клеммах батареи превышает ЭДС, поскольку \ (V = \ epsilon — Ir \) и I теперь отрицательны.

Рисунок \ (\ PageIndex {9} \): автомобильное зарядное устройство меняет нормальное направление тока через аккумулятор, обращая вспять его химическую реакцию и пополняя ее химический потенциал.

Важно понимать последствия внутреннего сопротивления источников ЭДС, таких как батареи и солнечные элементы, но часто анализ цепей выполняется с помощью напряжения на клеммах батареи, как мы делали в предыдущих разделах. Напряжение на клеммах обозначается просто как В , без индекса «клемма».Это связано с тем, что внутреннее сопротивление батареи трудно измерить напрямую, и оно может со временем измениться.

Авторы и авторство

Сэмюэл Дж. Линг (Государственный университет Трумэна), Джефф Санни (Университет Лойола Мэримаунт) и Билл Мобс со многими авторами. Эта работа лицензирована OpenStax University Physics в соответствии с лицензией Creative Commons Attribution License (4.0).

Физика — Электродвижущая сила — Бирмингемский университет

Электродвижущая сила (ЭДС) равна разности потенциалов на клеммах при отсутствии тока.ЭДС и разность потенциалов на клеммах ( В, ) измеряются в вольтах, но это не одно и то же. ЭДС ( ϵ ) — это количество энергии ( E ), обеспечиваемое батареей на каждый кулон заряда ( Q ), проходящий через нее.

Как рассчитать ЭДС?

ЭДС можно записать через внутреннее сопротивление батареи ( r ) где: ϵ = I (r + R )

Что из закона Ома, мы можем затем изменить это с точки зрения оконечного сопротивления: ϵ = В + Ir

ЭДС ячейки может быть определена путем измерения напряжения на ячейке с помощью вольтметра и тока в цепи с помощью амперметра для различных сопротивлений.Затем мы можем настроить схему для определения ЭДС, как показано ниже.

ЭДС и внутреннее сопротивление электрических элементов и батарей

Исследование ЭМП

Как закон Фарадея соотносится с ЭМП?

Закон Фарадея гласит, что любое изменение магнитного поля катушки будет индуцировать в катушке ЭДС (а следовательно, и ток). Он пропорционален минус скорости изменения магнитного потока ( ϕ ) (примечание N — количество витков в катушке).

Согласно закону Фарадея, общество извлекло выгоду из таких важных технологий, как трансформаторы, которые используются для передачи электроэнергии в национальной энергосистеме Великобритании, которая сейчас необходима в наших домах. Также он используется в электрических генераторах и двигателях, таких как плотины гидроэлектростанций, которые производят электричество, которое сейчас является неотъемлемой частью наших современных технологических потребностей. Текущий исследовательский проект MAG-DRIVE в Бирмингеме направлен на поиск способов разработки и улучшения материалов с постоянными магнитами, которые могут быть использованы в электромобилях следующего поколения.ЭМП также генерируется солнечными батареями, поэтому они важны для исследований в области возобновляемых источников энергии.

Лабораторные признания

Исследователи подкаста In the Laboratory Confessions рассказывают о своем лабораторном опыте в контексте практических экзаменов A Level. Эпизоды, посвященные правильному использованию цифровых инструментов (простое гармоническое движение), правильному построению принципиальных схем (удельное сопротивление в проводе) и использованию источников питания постоянного тока (конденсаторов), имеют отношение к эксперименту по ЭДС, ниже вы можете услышать удельное сопротивление. в проводном подкасте.

Как мы интерпретируем наши данные?

По мере увеличения сопротивления переменного резистора величина тока будет уменьшаться. График зависимости напряжения от тока должен давать линейную зависимость, где градиент линии дает отрицательное внутреннее сопротивление ячейки ( -r ), а точка пересечения дает ЭДС (напряжение, при котором ток равен 0).

Выполнение нескольких измерений при разных значениях сопротивления даст больше точек на графике V-I, что сделает подбор более надежным.Также рекомендуется повторить измерения, так как ячейка будет постепенно стекать, что повлияет на показания. Во избежание разряда элемента / батареи ее следует отключать между измерениями. В качестве альтернативы в схему можно включить выключатель. Также не рекомендуется использовать аккумуляторные батареи, так как они имеют низкое внутреннее сопротивление.

Несмотря на то, что этот эксперимент довольно прост, он поможет вам отличить конечную разницу от ЭДС, что может быть сложной концепцией для понимания учащимися.Поскольку люди все больше полагаются на электричество, исследования, связанные с ЭМП, важны для развития и технического прогресса электричества.

Следующие шаги

Эти ссылки предоставляются только для удобства и в информационных целях; они не означают одобрения или одобрения Бирмингемским университетом какой-либо информации, содержащейся на внешнем веб-сайте. Бирмингемский университет не несет ответственности за точность, законность или содержание внешнего сайта или последующих ссылок.Пожалуйста, свяжитесь с внешним сайтом для получения ответов на вопросы относительно его содержания.

ЭДС и напряжение на клеммах

Когда вы забываете выключить автомобильные фары, они медленно тускнеют по мере разрядки аккумулятора. Почему они просто не мигают, когда батарея разряжена? Их постепенное затемнение означает, что выходное напряжение батареи уменьшается по мере разряда батареи. Причина снижения выходного напряжения для разряженных или перегруженных батарей заключается в том, что все источники напряжения состоят из двух основных частей — источника электрической энергии и внутреннего сопротивления.

Электродвижущая сила

Все источники напряжения создают разность потенциалов и могут подавать ток, если подключены к сопротивлению. В небольшом масштабе разность потенциалов создает электрическое поле, которое воздействует на заряды, вызывая ток. Мы называем эту разность потенциалов электродвижущей силой (сокращенно ЭДС). ЭДС — это вообще не сила; это особый тип разности потенциалов источника при отсутствии тока. Единицы измерения ЭДС — вольты.

Электродвижущая сила напрямую связана с источником разности потенциалов, например с конкретной комбинацией химических веществ в батарее.Однако при протекании тока ЭДС отличается от выходного напряжения устройства. Напряжение на выводах батареи, например, меньше, чем ЭДС, когда батарея подает ток, и оно падает дальше, когда батарея разряжается или разряжается. Однако, если выходное напряжение устройства можно измерить без потребления тока, то выходное напряжение будет равно ЭДС (даже для сильно разряженной батареи).

Напряжение на клеммах

представляет собой схематическое изображение источника напряжения.Выходное напряжение устройства измеряется на его выводах и называется напряжением на выводах В . Напряжение на клеммах определяется уравнением:

Схематическое изображение источника напряжения

Любой источник напряжения (в данном случае сухой углерод-цинковый элемент) имеет ЭДС, связанную с источником разности потенциалов, и внутреннее сопротивление r, связанное с его конструкцией. (Обратите внимание, что сценарий E означает ЭДС.) Также показаны выходные клеммы, на которых измеряется напряжение на клеммах V.Поскольку V = ЭДС-Ir, напряжение на клеммах равно ЭДС, только если ток не течет.

$ V = ЭДС — Ir $,

где r — внутреннее сопротивление, а I — ток, протекающий во время измерения.

I — положительный, если ток течет от положительного вывода. Чем больше ток, тем меньше напряжение на клеммах. Точно так же верно, что чем больше внутреннее сопротивление, тем меньше напряжение на клеммах.

домашних заданий и упражнений — Как соотношение напряжения на ЭДС показывает эффективность?

Ваш вопрос мог бы быть более ясным, используя контекст.Но я думаю, вы имеете в виду, что батарея подает в схему определенное значение $ \ mathrm {emf} $, а элементам схемы требуется определенное напряжение $ V $ для протекания тока.

Теперь, если напряжение $ V $ на всех элементах схемы (суммированное) на меньше, чем на , чем подаваемое $ \ mathrm {emf} $, то некоторые из них теряются. Это означает, что некоторые части схемы, которая была , а не , должны иметь сопротивление, а имеет сопротивление. Это могут быть, например, провода между элементами схемы.На таких нежелательных сопротивлениях будет падение напряжения, и если я прав, предполагая, что эти падения напряжения не включены в напряжение $ V $, о котором вы говорите, тогда отношение $ \ mathrm {emf} $ к $ V $ просто сообщает вам , сколько было потеряно из-за таких нежелательных сопротивлений:

$$ \ eta = \ frac {V} {\ mathrm {emf}} $$

  • Если $ \ eta = 1 $, то $ V $ равно $ \ mathrm {emf} $, и все поставленные $ \ mathrm {emf} $ были «потрачены», как и предполагалось. Это эффективность $ 100 \, \% $, когда вся добавленная энергия (на коломб) расходуется полезным (предполагаемым) образом.
  • Если $ \ eta <1 $, тогда $ V $ на меньше, чем на $ \ mathrm {emf} $, и некоторые из них были «потеряны» из-за нежелательных сопротивлений. Тогда ваша эффективность будет меньше 100 $ \, \% $, поскольку менее $ 100 \, \% $ энергии (на столбец), добавленной в схему, было потрачено полезным (предполагаемым) способом.

В определенных цепях, где должна выполняться определенная «работа» — например, в фонарике, где определенное сопротивление цепи должно преобразовывать добавленную электрическую энергию в свет, или в тостере, где определенное сопротивление должно преобразовывать добавленную электрическую энергию в тепло — тогда у вас может быть КПД, определяемый как энергии, используемой для этой конкретной «работы», деленной на энергию (на столбец), подаваемую от батареи .Затем внезапно возникают потери не только из-за нежелательных малых сопротивлений в проводах и т. Д., Но и во всех других элементах схемы, помимо сопротивления нагрева, которые необходимы для работы схемы. Тогда эффективность покажет вам, насколько велика доля энергии (на столбец), которая используется специально для «работы» или цели устройства / цепи.

Убедитесь, что напряжение $ V $ покрывает, когда вы подключаете его к формуле — иначе вы не сможете узнать, что вам говорит эффективность.

ЭДС и внутреннее сопротивление

ЭДС и внутреннее сопротивление
следующий: резисторы последовательно и Up: Electric Current Предыдущий: Сопротивление и удельное сопротивление Теперь настоящие батареи изготавливаются из материалов с ненулевым удельным сопротивлением.Отсюда следует, что настоящие батареи — это не просто источники чистого напряжения. Они также обладают внутренние сопротивления . Между прочим, чистое напряжение Источник обычно обозначается как ЭДС (что означает электродвижущую силу ). Конечно, ЭДС измеряется в вольтах. Аккумулятор можно смоделировать как ЭДС, включенную последовательно с резистором. , который представляет собой его внутреннее сопротивление. Предположим, что такие батарея используется для управления током через внешний нагрузочный резистор, так как изображенный на рис.17. Обратите внимание, что на принципиальных схемах ЭДС представлена ​​в виде двух близко расположенных параллельных линии неравной длины. Электрический потенциал более длинной линии больше, чем тот из более коротких по вольтам. Резистор представлен как зигзагообразная линия.
Рисунок 17: Батарея ЭДС и внутреннего сопротивления подключена к нагрузочному резистору сопротивления.

Рассмотрим батарею на рисунке.Напряжение аккумулятора равно определяется как разница в электрическом потенциале между его положительным и отрицательные клеммы: т.е. , точки и соответственно. Когда мы переходим от к , электрический потенциал увеличивается на вольт, когда мы пересекаем ЭДС, но затем уменьшается на вольт, когда мы пересекаем внутренний резистор. Падение напряжения на резисторе следует из закона Ома, из которого следует, что падение напряжения на резисторе, несущем ток , находится в том направлении, в котором текущие потоки.Таким образом, напряжение аккумулятора связано с его ЭДС. и внутреннее сопротивление через

(133)

Обычно мы думаем, что ЭДС батареи по существу постоянная (поскольку она зависит только от химической реакции, происходящей внутри батареи, которая преобразует химическая энергия в электрическую), поэтому мы должны заключить, что напряжение батарея на самом деле уменьшается по мере увеличения тока, потребляемого от нее.Фактически, напряжение равно только ЭДС при пренебрежимо малом токе. Текущий розыгрыш от аккумулятора обычно не может превышать критического значения
(134)

поскольку напряжение становится отрицательным (что может произойти только если резистор нагрузки также отрицательный: это практически невозможно). Отсюда следует, что если мы закоротим аккумулятор, подключив его положительные и отрицательные клеммы вместе с использованием проводящего провода с незначительным сопротивлением, ток, потребляемый от батареи, ограничен ее внутренним сопротивлением.Фактически в этом случае сила тока равна максимально возможной. Текущий .

Настоящая батарея обычно характеризуется его ЭДС ( т.е. , его напряжение при нулевом токе) и максимальный ток, который он может подавать. Например, стандартный сухой элемент (, т.е. , своего рода аккумулятор, используемый для питания калькуляторов и фонарей) обычно рассчитан на и скажи) . Таким образом, ничего действительно катастрофического не произойдет. происходит, если мы закорачиваем сухой элемент.Мы разрядим аккумулятор через сравнительно короткий промежуток времени, но опасно большой ток не будет поток. С другой стороны, автомобильный аккумулятор обычно рассчитывается на и что-то вроде (такой ток нужен для запустить стартер). Понятно, что автомобильный аккумулятор должен иметь много более низкое внутреннее сопротивление, чем у сухого элемента. Отсюда следует, что если мы были достаточно глупы, чтобы замкнуть автомобильный аккумулятор, в результате довольно катастрофически (представьте себе всю энергию, необходимую для запуска двигателя автомобиль собирается тонким проводом, соединяющим клеммы аккумулятора вместе).



следующий: резисторы последовательно и Up: Electric Current Предыдущий: Сопротивление и удельное сопротивление
Ричард Фицпатрик 2007-07-14

Электроэнергия — Веб-формулы

Электрическая мощность определяется по формуле:
P = V · I
Где V — напряжение, а I — ток.

Соответствующие единицы:
ватт (Вт) = вольт (В) · ампер (A)


Мощность также можно определить по следующим формулам:
P = I 2 · R R = P / I 2 I R )
P = V 2 / R R = V 2 9038 9038 V = √ ( P · R )


Подробнее об Electric Power 9030 4
Электроэнергия определяется как скорость, с которой работа выполняется источником эл.м.ф. в поддержании тока в электрической цепи. Практическая единица мощности — киловатт и лошадиные силы; где 1 киловатт = 100 ватт и 1 л.с. = 746 ватт.

Если сопротивления (например, электрические приборы) соединены последовательно, ток через каждое сопротивление будет одинаковым. Тогда мощность электрического прибора, P α R и P α V (поскольку V = IR), это означает, что в последовательной комбинации сопротивлений разность потенциалов и потребляемая мощность будут больше при большем сопротивлении .

Если сопротивления ( i.е. электроприборов) подключены параллельно, разность потенциалов на каждом приборе одинакова. Тогда P α 1 / R и I α 1 / R (как V = IR), что означает, что в параллельной комбинации сопротивлений потребляемый ток и мощность будут больше при меньшем сопротивлении.

Для данного напряжения В, , если сопротивление изменяется с R на ( R / n ), а потребляемая мощность изменяется с P на nP , затем согласно P = V 2 / R , имеем:


P = V 2 / (R / n)) = n (V 2 / R) = nP, где R = R / n и P = nP

Когда приборы питания P 1 , P 2 , P 3 P n включены последовательно с источником напряжения, эффективная потребляемая мощность ( P s ) определяется по формуле:


1/ P s = 1 / P 1 + 1 / P 2 + 1 / P 3 +… + 1 / P n
Для n приборов, каждый из сопротивление R , соединены последовательно с источником напряжения В, рассеиваемая мощность P s тогда задается как:
(1) P s = V 2 / n R

Когда приборы питания

P 1 , P 2 , P 3 P n подключены параллельно к источнику напряжения, эффективная мощность потреблено ( P p ) затем определяется по:
P s = P 1 + P 2 + P 3 +… + P n Для приборов n , каждое с равным сопротивлением R , подключено параллельно к источнику напряжения В , рассеиваемая мощность тогда определяется как:
(2) P p = В 2 / ( R / n) = n V 2 / R

Из (1) и (2) мы имеем P p / P s = n 2 или просто записывается как : P P = n 2 P s .

В соответствии с приведенными выше формулами, мы можем объяснить, что:


При группировке лампочек серии по одному источнику напряжения, лампа с большей мощностью будет давать меньшую яркость и будет иметь меньший потенциал сопротивления, но тот же ток. , тогда как в параллельном группировании лампочек по данному источнику напряжения лампа большей мощности даст большую яркость и позволит большему току проходить через нее, но будет иметь меньшее сопротивление и такую ​​же разность потенциалов на нем.

Электроэнергия
Электроэнергия определяется как общая выполненная работа или энергия, поставленная источником ЭДС. при поддержании тока в электрической цепи в течение заданного времени:
Электрическая энергия = электрическая мощность × время = P × t

Таким образом, формула для электрической энергии имеет вид:
Электрическая энергия = P × t = V × I × t = I 2 × R × t = V 2 t / R

S.I единица электрической энергии — джоуль (обозначается J), где 1 джоуль = 1 ватт × 1 секунда = 1 вольт × 1 ампер × 1 секунда
Коммерческая единица электрической энергии — киловатт-час ( кВт · ч, ), где 1 кВтч = 1000 Вт h = 3,6 × 10 6 J = одна единица потребляемой электроэнергии .

Количество единиц потребляемой электроэнергии равно n = (общая мощность × время в часе) / 1000
Стоимость потребления электроэнергии в доме = количество.единиц потребленной электроэнергии × количество на одну единицу электроэнергии.

Максимальная мощность Теорема
В ней говорится, что выходная мощность источника тока максимальна, когда внутреннее сопротивление источника равно внешнему сопротивлению в цепи. Итак, если R — внешнее сопротивление цепи, а r — внутреннее сопротивление источника тока (то есть батареи), то выходная мощность максимальна, когда R = r.

Эта теорема применима ко всем типам источников ЭДС. и связан с выходной мощностью, а НЕ с рассеиваемой мощностью.

Если E — применяемая ЭДС. источника ЭДС. то есть . батарея с внутренним сопротивлением r и R — внешнее сопротивление, тогда ток в цепи определяется как:
I = E / (R + r)

При максимальной выходной мощности R = r , поэтому имеем:
I = E / (r + r) = E / (2r)
и
максимальная выходная мощность:
P max = I 2 r = E 2 / (4r)

При коротком замыкании аккумулятора мощность равна нулю.В этом случае вся мощность батареи рассеивается внутри батареи из-за ее внутреннего сопротивления. Таким образом, мощность, рассеиваемая внутри батареи, определяется как: P = ( E / r) 2 × r = E 2 / r

КПД источника ЭДС.
КПД источника ЭДС. определяется как отношение выходной мощности (, т. е. , мощность на внешнем сопротивлении цепи, к входной мощности (т. е.мощность, потребляемая от источника ЭДС). Итак,

Где V = падение потенциала на внешнем сопротивлении R,
E = E.M.F. источника тока,
I = ток в цепи.

Если r — внутреннее сопротивление источника ЭДС, тогда
В = IR и E = I (R + r )
или

Когда мощность, полученная от источника, максимальна, тогда R = р. В данной ситуации имеем:

Таким образом, максимальная эффективность источника эл.м.ф. составляет 50%. Это означает, что для элемента только половина общей мощности, потребляемой от элемента, используется для полезных целей, тогда как другая половина рассеивается внутри элемента.

Пример 1:
Лифт должен поднимать 1000 кг на расстояние 100 м со скоростью 4 м / с. Какую в среднем мощность оказывает лифт во время этой поездки?
Решение:
Работу, проделанную лифтом на 100 метров, легко вычислить:
W = mgh = (1000) (9.8) (100) = 9,8 × 10 5 Джоулей.

Общее время поездки можно рассчитать по скорости лифта:
t = x / v = 100 м / 4 м / с = 25 с .

Таким образом, средняя мощность определяется по формуле: P = Вт / t = 9,8 × 10 5 / 25s = 3,9 × 10 4 Вт или 39 кВт.

Пример 2:
Считается, что объект в свободном падении достиг конечной скорости , если сопротивление воздуха становится достаточно сильным, чтобы противодействовать всему ускорению свободного падения, заставляя объект падать с постоянной скоростью.Точное значение конечной скорости зависит от формы объекта, но для многих объектов оно может быть оценено на уровне 100 м / с. Когда объект весом 10 кг достиг предельной скорости, какую силу сопротивление воздуха оказывает на объект?

Решение: Для решения этой проблемы мы будем использовать уравнение P = Fv cos θ , Вместо обычного уравнения мощности, поскольку нам дана скорость объекта. Нам просто нужно вычислить силу, прилагаемую к объекту сопротивлением воздуха, и угол между силой и скоростью объекта.Поскольку объект достиг постоянной скорости, результирующая сила, действующая на него, должна быть равна нулю. Поскольку на объект действуют только две силы: сила тяжести и сопротивление воздуха, сопротивление воздуха должно быть равным по величине и противоположным по направлению силе тяжести. Таким образом, F a = — F G = мг = 98 Н, направленным вверх. Таким образом, сила, прилагаемая сопротивлением воздуха, антипараллельна скорости объекта. Таким образом:
P = Fv cos θ = (98) (100) (cos180) = — 9800 Вт

Пример 3: Мощность двигателя насоса составляет 4 кВт.Сколько воды в кг / мин он может поднять на высоту 20 м? (g = 10 м / с 2 )
Решение:
Заданная мощность двигателя P = 4KW = 4000 Вт
Если масса воды, поднятая за одну секунду, = m кг.
Общий объем работы, выполненной при подъеме воды, W = mgh
Мощность P = Вт / т, но t = 1 минута = 60 сек.
4000 = mgh / 60
4000 = (m × 10 × 20) / 60
m = 1200 кг.

Пример 4 : Когда вода течет по трубе, ее скорость изменяется на 5%, найти изменение силы воды?
Решение: Мощность = Сила × Скорость = Скорость изменения количества движения × скорость = {(масса / время) × скорость} x скорость = {(adv) × v} × v = adv 3 где «a» — площадь поперечного сечения, «d» — плотность воды, а «v» — скорость потока воды.
Следовательно, Сила воды прямо пропорциональна кубу скорости воды, поэтому пусть
P = Kv 3 (k — постоянная величина, равная ad.)
Ведение журнала с обеих сторон
log P = 3log v + log k
Дифференциация с обеих сторон
dP / P = 3dv / v
процентное изменение мощности, dP / P × 100 = 3 × 5% = 15%.

Пример 5 : Кинетическая энергия выбрасываемой воды из плотины используется для вращения турбины. Труба, по которой устремляется вода — 2.4 метра и его скорость 12 м / сек. Предполагая, что вся кинетическая энергия воды используется для вращения турбины, вычислите производимый ток, если эффективность динамо-машины составляет 60% и станция передает мощность 240 кВ. Плотность воды = 10 3 кг / м 3 .
Решение: Учитывая, что
r = радиус трубы = 1,2 м, средняя скорость воды v = 12 м / с
V = 240 кВ = 240 × 10 3 вольт, плотность воды p = 10 3 кг / м 3 .
Итак, кинетическая энергия стремящейся воды в секунду, т.е.
Мощность P = (1/2) (массовый расход в секунду) × v 2
= (1/2) pr 2 (л / т) rv 2
= (1/2) pr 2 rv 3
= (1/2) 3,14 × (1,2) 2 × 10 3 × (12) 3 Вт
= 3,9 x 10 6 Вт

Ток в кабелях передачи определяется по формуле:
ток = выходная мощность / напряжение
= (60% мощности P) / (240 × 1000)
= [(60/100) × 3.9 × 10 6 ] / (240 × 1000) = 9,75 A

Оценка внутреннего сопротивления в цепях | Электрические схемы

Рабочий пример 7: Внутреннее сопротивление в цепи с последовательными резисторами

Для следующей схемы рассчитайте:

  1. разности потенциалов \ (V_ \ text {1} \), \ (V_ \ text {2} \) и \ (V_ \ text {3} \) на резисторах \ (R_ \ text {1} \), \ (R_ \ text {2} \) и \ (R_ \ text {3} \)

    .
  2. сопротивление \ (R_ \ text {3} \).

  3. сопротивление \ (R_ \ text {3} \).

Если внутреннее сопротивление равно \ (\ text {0,1} \) \ (\ text {Ω} \), какова ЭДС батареи и какая мощность рассеивается внутренним сопротивлением батареи?

Примечание

Это вопрос, очень похожий на то, что вы видели ранее. Это необходимо для того, чтобы выделить Дело в том, что подход к внутреннему сопротивлению строится на том же принципы, с которыми вы уже работали.

Определите, как подойти к проблеме

Нам дана разность потенциалов на ячейке и ток в цепи, а также сопротивления двух из трех резисторов. Мы можем использовать закон Ома для расчета разности потенциалов на известных резисторах. Поскольку резисторы включены в последовательную цепь, разность потенциалов равна \ (V = V_ \ text {1} + V_ \ text {2} + V_ \ text {3} \), и мы можем вычислить \ (V_ \ text {3} \). Теперь мы можем использовать эту информацию, чтобы найти разность потенциалов на неизвестном резисторе \ (R_ \ text {3} \).

Вычислить разность потенциалов на \ (R_ \ text {1} \)

Используя закон Ома: \ begin {align *} R_ \ text {1} & = \ frac {V_ \ text {1}} {I} \\ I \ cdot R_ \ text {1} & = I \ cdot \ frac {V_ \ text {1}} {I} \\ V_ \ text {1} & = {I} \ cdot {R_ \ text {1}} \\ & = 2 \ cdot 1 \\ V_ \ текст {1} & = \ текст {2} \ текст {V} \ end {align *}

Вычислить разность потенциалов на \ (R_ \ text {2} \)

Снова используя закон Ома: \ begin {align *} R_ \ text {2} & = \ frac {V_ \ text {2}} {I} \\ I \ cdot R_ \ text {2} & = I \ cdot \ frac {V_ \ text {2}} {I} \\ V_ \ text {2} & = {I} \ cdot {R_ \ text {2}} \\ & = 2 \ cdot 3 \\ V_ \ текст {2} & = \ текст {6} \ текст {V} \ end {align *}

Вычислить разность потенциалов на \ (R_ \ text {3} \)

Поскольку разность потенциалов на всех резисторах вместе взятых должна быть такой же, как разность потенциалов на ячейке в последовательной цепи, мы можем найти \ (V_ \ text {3} \), используя: \ begin {align *} V & = V_ \ text {1} + V_ \ text {2} + V_ \ text {3} \\ V_ \ text {3} & = V — V_ \ text {1} — V_ \ text {2} \\ & = 23-2-6 \\ V_ \ текст {3} & = \ текст {15} \ текст {V} \ end {align *}

Найдите сопротивление \ (R_ \ text {3} \)

Нам известна разность потенциалов на \ (R_ \ text {3} \) и ток через нее, поэтому мы можем использовать закон Ома для вычисления значения сопротивления: \ begin {align *} R_ \ text {3} & = \ frac {V_ \ text {3}} {I} \\ & = \ frac {\ text {15}} {\ text {2}} \\ R_ \ text {3} & = \ text {7,5} ~ ​​\ Omega \ end {align *}

Разница потенциалов на внутреннем сопротивлении батареи

Значение ЭДС можно рассчитать по разности потенциалов нагрузки и разности потенциалов на внутреннем сопротивлении.2} {R} \), и мы знаем ток в цепи, внутреннее сопротивление и разность потенциалов на нем, поэтому мы можем использовать любую форму уравнения для мощности:

\ begin {align *} P_r & = V_rI_r ​​\\ & = (\ текст {0,2}) (\ текст {2}) \\ & = \ текст {0,4} \ текст {W} \ end {align *}

Напишите окончательный ответ

  • \ (V_ \ text {1} = \ text {2,0} \ text {V} \)
  • \ (V_ \ text {2} = \ text {6,0} \ text {V} \)
  • \ (V_ \ text {3} = \ text {10,0} \ text {V} \)
  • \ (R_ \ text {3} = \ text {7,5} \ Omega \)
  • \ (\ mathcal {E} = \ text {23,2} \ text {V} \)
  • \ (P_r = \ text {0,4} \ text {W} \)

Рабочий пример 8: Внутреннее сопротивление и резисторы параллельно

Разность потенциалов на батарее составляет 18 В, когда она подключена к двум параллельным резисторам \ (\ text {4,00} \) \ (\ Omega \) и \ (\ text {12,00} \) \ ( \ Omega \) соответственно.Рассчитайте ток через ячейку и через каждый из резисторов. Если внутреннее сопротивление батареи \ (\ text {0,375} \) \ (\ text {Ω} \), какова ЭДС батареи?

Сначала нарисуйте схему, прежде чем производить какие-либо вычисления

Определите, как подойти к проблеме

Нам нужно определить ток через ячейку и каждый из параллельных резисторов. Нам дана разность потенциалов на ячейке и сопротивления резисторов, поэтому мы можем использовать закон Ома для расчета тока.

Рассчитать ток через ячейку

Чтобы рассчитать ток через элемент, нам сначала нужно определить эквивалентное сопротивление остальной части цепи. Резисторы включены параллельно и поэтому: \ begin {align *} \ frac {\ text {1}} {R} & = \ frac {\ text {1}} {R_ \ text {1}} + \ frac {\ text {1}} {R_ \ text {2}} \ \ & = \ frac {\ text {1}} {\ text {4}} + \ frac {\ text {1}} {\ text {12}} \\ & = \ frac {3 + 1} {\ text {12}} \\ & = \ frac {\ text {4}} {\ text {12}} \\ R & = \ frac {\ text {12}} {\ text {4}} = \ text {3,00} \ \ Omega \ end {выровнять *} Теперь, используя закон Ома, чтобы найти ток через ячейку: \ begin {align *} R & = \ frac {V} {I} \\ I & = \ frac {V} {R} \\ & = \ frac {\ text {18}} {\ text {3}} \\ I & = \ text {6,00} \ text {A} \ end {align *}

Теперь определите ток через один из параллельных резисторов

Мы знаем, что для чисто параллельной конфигурации резисторов разность потенциалов на ячейке такая же, как и разность потенциалов на каждом из параллельных резисторов.Для этой схемы: \ begin {align *} V & = V_ \ text {1} = V_ \ text {2} = \ text {18} \ text {V} \ end {выровнять *} Начнем с вычисления тока через \ (R_ \ text {1} \) по закону Ома: \ begin {align *} R_ \ text {1} & = \ frac {V_ \ text {1}} {I_ \ text {1}} \\ I_ \ text {1} & = \ frac {V_ \ text {1}} {R_ \ text {1}} \\ & = \ frac {\ text {18}} {\ text {4}} \\ I_ \ text {1} & = \ text {4,50} \ text {A} \ end {align *}

Рассчитайте ток через другой параллельный резистор

Мы можем снова использовать закон Ома, чтобы найти ток в \ (R_ \ text {2} \): \ begin {align *} R_ \ text {2} & = \ frac {V_ \ text {2}} {I_ \ text {2}} \\ I_ \ text {2} & = \ frac {V_ \ text {2}} {R_ \ text {2}} \\ & = \ frac {\ text {18}} {\ text {12}} \\ I_ \ text {2} & = \ text {1,50} \ text {A} \ end {выровнять *} Альтернативный метод вычисления \ (I_ \ text {2} \) заключался бы в использовании того факта, что токи через каждый из параллельных резисторов должны составлять суммарный ток через ячейку: \ begin {align *} I & = I_ \ text {1} + I_ \ text {2} \\ I_ \ text {2} & = I — I_ \ text {1} \\ & = 6 — 4.5 \\ I_ \ text {2} & = \ text {1,5} \ text {A} \ end {align *}

Определить ЭДС

Суммарный ток через батарею — это ток через внутреннее сопротивление батареи. Знание силы тока и сопротивления позволяет нам использовать закон Ома для определения разности потенциалов на внутреннем сопротивлении и, следовательно, ЭДС батареи.

Используя закон Ома, мы можем определить разность потенциалов на внутреннем сопротивлении:

\ begin {align *} V & = I \ cdot r \\ & = \ текст {6} \ cdot \ text {0,375} \\ & = \ текст {2,25} \ текст {V} \ end {выровнять *}

Мы знаем, что ЭДС аккумулятора — это разность потенциалов на выводе, суммированная с разностью потенциалов на внутреннем сопротивлении, поэтому:

\ begin {align *} \ mathcal {E} & = V + Ir \\ & = \ text {18} + \ text {2,25} \\ & = \ текст {20,25} \ текст {V} \ end {align *}

Напишите окончательный ответ

Ток через ячейку равен \ (\ text {6,00} \) \ (\ text {A} \).

Ток через резистор \ (\ text {4,00} \) \ (\ Omega \) равен \ (\ text {4,50} \) \ (\ text {A} \).

Ток через резистор \ (\ text {12,00} \) \ (\ Omega \) равен \ (\ text {1,50} \) \ (\ text {A} \).

ЭДС аккумулятора равна \ (\ text {20,25} \) \ (\ text {V} \).

Рабочий пример 9: Мощность в последовательной и параллельной сетях резисторов

Учитывая следующую схему:

Ток, покидающий батарею, равен \ (\ text {1,07} \) \ (\ text {A} \), общая мощность, рассеиваемая во внешней цепи, равна \ (\ text {6,42} \) \ ( \ text {W} \), отношение полных сопротивлений двух параллельных сетей \ (R_ {P \ text {1}}: R_ {P \ text {2}} \) равно 1: 2, соотношение \ (R_ \ text {1}: R_ \ text {2} \) равно 3: 5 и \ (R_ \ text {3} = \ text {7,00} \ text {Ω} \).

Определите:

  1. разность потенциалов АКБ,
  2. мощность, рассеиваемая в \ (R_ {P \ text {1}} \) и \ (R_ {P \ text {2}} \), и
  3. , если батарея имеет ЭДС, равную \ (\ text {6,50} \) \ (\ text {V} \), каково значение сопротивления каждого резистора и мощность, рассеиваемая в каждом из них.

Что требуется

В этом вопросе вам дается различная информация и предлагается определить мощность, рассеиваемую на каждом резисторе и каждой комбинации резисторов.Обратите внимание, что данная информация в основном относится ко всей цепи. Это подсказка, которую вы должны начать с общей схемы и двигаться вниз к более конкретным элементам схемы.

Расчет разности потенциалов аккумулятора

В первую очередь остановимся на батарее. Нам дана мощность всей цепи, а также ток, покидающий батарею. Мы знаем, что разность потенциалов на клеммах аккумулятора — это разность потенциалов в цепи в целом.

Мы можем использовать соотношение \ (P = VI \) для всей схемы, потому что разность потенциалов такая же, как разность потенциалов на клеммах батареи: \ begin {align *} P & = VI \\ V & = \ frac {P} {I} \\ & = \ frac {\ text {6,42}} {\ text {1,07}} \\ & = \ текст {6,00} \ текст {V} \ end {align *}

Разность потенциалов на батарее равна \ (\ text {6,00} \) \ (\ text {V} \).

Мощность, рассеиваемая в \ (R_ {P \ text {1}} \) и \ (R_ {P \ text {2}} \)

Помните, что мы работаем от общих деталей схемы вниз к деталям отдельных элементов, это противоположно тому, как вы относились к этой схеме ранее.

Мы можем рассматривать параллельные сети как эквивалентные резисторы, поэтому схема, с которой мы сейчас работаем, будет выглядеть так:

Мы знаем, что ток через два элемента схемы будет одинаковым, потому что это последовательная цепь и что сопротивление всей цепи должно быть: \ (R_ {Ext} = R_ {P \ text {1}} + R_ {P \ text {2}} \). Мы можем определить полное сопротивление по закону Ома для цепи в целом: \ begin {align *} V_ {батарея} & = IR_ {Ext} \\ R_ {Ext} & = \ frac {V_ {аккумулятор}} {I} \\ & = \ frac {\ text {6,00}} {\ text {1,07}} \\ & = \ текст {5,61} \ текст {Ω} \ end {align *}

Мы знаем, что соотношение между \ (R_ {P \ text {1}}: R_ {P \ text {2}} \) равно 1: 2, что означает, что мы знаем: \ begin {align *} R_ {P \ text {1}} & = \ frac {\ text {1}} {\ text {2}} R_ {P \ text {2}} \ \ \ text {и} \\ R_T & = R_ {P \ text {1}} + R_ {P \ text {2}} \\ & = \ frac {\ text {1}} {\ text {2}} R_ {P \ text {2}} + R_ {P \ text {2}} \\ & = \ frac {\ text {3}} {\ text {2}} R_ {P \ text {2}} \\ (\ text {5,61}) & = \ frac {\ text {3}} {\ text {2}} R_ {P \ text {2}} \\ R_ {P \ text {2}} & = \ frac {\ text {2}} {\ text {3}} (\ text {5,61}) \\ R_ {P \ text {2}} & = \ text {3,74} \ text {Ω} \ end {выровнять *} и поэтому: \ begin {align *} R_ {P \ text {1}} & = \ frac {\ text {1}} {\ text {2}} R_ {P \ text {2}} \\ & = \ frac {\ text {1}} {\ text {2}} (3.2 (\ text {3,74}) \\ & = \ текст {4,28} \ текст {W} \ end {выровнять *} Эти значения будут в сумме с исходным значением мощности, которое у нас было для внешней цепи. Если бы они не мы бы сделали ошибку в расчетах.

Расчет параллельной сети 1

Теперь мы можем приступить к детальному расчету первого набора параллельных резисторов.

Мы знаем, что соотношение между \ (R _ {\ text {1}}: R _ {\ text {2}} \) составляет 3: 5, что означает, что мы знаем \ (R _ {\ text {1}} = \ frac {\ text {3}} {\ text {5}} R _ {\ text {2}} \).Нам также известно общее сопротивление двух параллельных резисторов в этой сети. это \ (\ text {1,87} \) \ (\ text {Ω} \). Мы можем использовать соотношение между значениями двух резисторов, а также формула для общей сопротивление (\ (\ frac {\ text {1}} {R_PT} = \ frac {\ text {1}} {R_ \ text {1}} + \ frac {\ text {1}} {R_ \ text {2) }} \)) чтобы найти номиналы резисторов: \ begin {align *} \ frac {\ text {1}} {R_ {P \ text {1}}} & = \ frac {\ text {1}} {R_ \ text {1}} + \ frac {\ text {1}} { R_ \ text {2}} \\ \ frac {\ text {1}} {R_ {P \ text {1}}} & = \ frac {\ text {5}} {3R_ \ text {2}} + \ frac {\ text {1}} { R_ \ text {2}} \\ \ frac {\ text {1}} {R_ {P \ text {1}}} & = \ frac {\ text {1}} {R_ \ text {2}} (\ frac {\ text {5}} { \ text {3}} + 1) \\ \ frac {\ text {1}} {R_ {P \ text {1}}} & = \ frac {\ text {1}} {R_ \ text {2}} (\ frac {\ text {5}} { \ text {3}} + \ frac {\ text {3}} {\ text {3}}) \\ \ frac {\ text {1}} {R_ {P \ text {1}}} & = \ frac {\ text {1}} {R_ \ text {2}} \ frac {\ text {8}} {\ текст {3}} \\ R_ \ text {2} & = R_ {P \ text {1}} \ frac {\ text {8}} {\ text {3}} \\ & = (\ text {1,87}) \ frac {\ text {8}} {\ text {3}} \\ & = \ текст {4,99} \ текст {Ω} \ end {выровнять *} Мы также можем вычислить \ (R _ {\ text {1}} \): \ begin {align *} R _ {\ text {1}} & = \ frac {\ text {3}} {\ text {5}} R _ {\ text {2}} \\ & = \ frac {\ text {3}} {\ text {5}} (\ text {4,99}) \\ & = \ текст {2,99} \ текст {Ω} \ end {align *}

Для определения мощности нам нужно рассчитанное нами сопротивление и либо разность потенциалов, либо ток. 2} {\ text {4,99}} \\ & = \ текст {0,80} \ текст {W} \ end {align *}

Параллельная сеть 2 расчета

Теперь мы можем приступить к детальному расчету второго набора параллельных резисторов.

Нам дано \ (R_ \ text {3} = \ text {7,00} \ text {Ω} \), и мы знаем \ (R_ {P \ text {2}} \), поэтому мы можем вычислить \ (R_ \ text {4} \) из: \ begin {align *} \ frac {\ text {1}} {R_ {P \ text {2}}} & = \ frac {\ text {1}} {R_ \ text {3}} + \ frac {\ text {1}} { R_ \ text {4}} \\ \ frac {\ text {1}} {\ text {3,74}} & = \ frac {\ text {1}} {\ text {7,00}} + \ frac {\ text {1}} {R_ \ текст {4}} \\ R_ \ text {4} & = \ text {8,03} \ text {Ω} \ end {align *}

Мы можем вычислить разность потенциалов во второй параллельной сети, вычтя разность потенциалов первой параллельной сети из разности потенциалов батареи, \ (V_ {P \ text {2}} = \ text {6,00} — \ text {2,00} = \ text {4,00} \ text {V} \).2} {\ text {8,03}} \\ & = \ текст {1,99} \ текст {W} \ end {align *}

Внутреннее сопротивление

Мы знаем, что ЭДС батареи равна \ (\ text {6,5} \) \ (\ text {V} \), но что разность потенциалов, измеренная на клеммах, составляет всего \ (\ text {6} \) \ (\ текст {V} \). Разница — это разность потенциалов на внутреннем сопротивлении батареи, и мы можем использовать известный ток и закон Ома для определения внутреннего сопротивления:

\ begin {align *} V & = I \ cdot R \\ R & = \ frac {V} {I} \\ & = \ frac {\ text {0,5}} {\ text {1,07}} \\ & = \ text {0,4672897} \\ & = \ текст {0,47} \ текст {Ω} \ end {выровнять *}

Мощность, рассеиваемая внутренним сопротивлением батареи:

\ begin {align *} P & = VI \\ & = \ текст {0,5} \ cdot \ text {1,07} \\ & = \ текст {0,535} \ текст {W} \ end {align *}

Рабочий пример 10: Внутреннее сопротивление и фары [NSC 2011 Paper 1]

Фара и два ИДЕНТИЧНЫХ задних фонаря скутера подключены параллельно к батарее с неизвестным внутренним сопротивлением, как показано на упрощенной принципиальной схеме ниже.Фара имеет сопротивление \ (\ text {2,4} \) \ (\ text {Ω} \) и управляется переключателем \ (\ textbf {S} _1 \). Задние фонари управляются переключателем \ (\ textbf {S} _2 \). Сопротивлением соединительных проводов можно пренебречь.

График рядом показывает разность потенциалов на клеммах батареи до и после включения переключателя \ (\ textbf {S} _1 \) (пока переключатель \ (\ textbf {S} _2 \) открыт). Переключатель \ (\ textbf {S} _1 \) закрывается в момент \ (\ textbf {t} _1 \).

  1. Используйте график, чтобы определить ЭДС аккумулятора.

    (1 балл)

  2. ПРИ ТОЛЬКО ВЫКЛЮЧАТЕЛЬ \ (\ textbf {S} _1 \) ЗАКРЫТО, рассчитайте следующее:

    1. Ток через фару

      (3 балла)

    2. Внутреннее сопротивление \ (r \) батареи

      (3 балла)

  3. ОБЕ ПЕРЕКЛЮЧАТЕЛИ \ (\ textbf {S} _1 \) И \ (\ textbf {S} _2 \) ТЕПЕРЬ ЗАКРЫТЫ.В течение этого периода аккумулятор обеспечивает ток \ (\ text {6} \) \ (\ text {A} \).

    Рассчитайте сопротивление каждого заднего фонаря.

    (5 баллов)

  4. Как повлияет на показания вольтметра, если фара перегорит? (Оба переключателя \ (\ textbf {S} _1 \) и \ (\ textbf {S} _2 \) все еще закрыты.)

    Запишите только УВЕЛИЧИВАЕТ, УМЕНЬШАЕТСЯ или ОСТАЕТСЯ ОДИН ТО ЖЕ.

    Дайте объяснение.

    (3 балла)

Вопрос 1

\ (\ text {12} \) \ (\ text {V} \)

(1 балл)

Вопрос 2.1

Вариант 1:

\ begin {align *} I & = \ frac {V} {R} \\ & = \ frac {\ text {9,6}} {\ text {2,4}} \\ & = \ текст {4 A} \ end {выровнять *}

Вариант 2:

\ begin {align *} \ text {emf} & = IR + Ir \\ 12 & = I (\ text {2,4}) + \ text {2,4} \\ \ поэтому I & = \ text {4 A} \ end {выровнять *}

(3 балла)

Вопрос 2.2

Вариант 1:

\ begin {align *} \ text {emf} & = IR + Ir \\ 12 & = \ text {9,4} + 4r \\ r & = \ текст {0,6} \ \ Omega \ end {выровнять *}

Вариант 2:

\ begin {align *} V_ {потеряно} & = Ir \\ \ text {2,4} & = \ text {4} r \\ \ поэтому r & = \ text {0,6} \ \ Omega \ end {выровнять *}

Вариант 3:

\ begin {align *} \ text {emf} & = I (R + r) \\ \ text {12} & = \ text {4} (\ text {2,4} + r) \\ \ поэтому r & = \ text {0,6} \ \ Omega \ end {выровнять *}

(3 балла)

Вопрос 3

Вариант 1:

\ begin {align *} \ text {emf} & = IR + Ir \\ \ text {12} & = \ text {6} (R + \ text {0,6}) \\ R _ {\ text {ext}} & = \ text {1,4} \ \ Omega \ end {align *} \ begin {align *} \ frac {1} {R} & = \ frac {1} {R_ {1}} + \ frac {1} {R_ {2}} \\ \ frac {1} {\ text {1,4}} & = \ frac {1} {\ text {2,4}} + \ frac {1} {R} \\ R & = \ текст {3,36} \ \ Omega \ end {выровнять *}

Каждый задний фонарь: \ (R = \ text {1,68} \ \ Omega \)

Вариант 2:

\ begin {align *} \ text {Emf} & = V _ {\ text {terminal}} + Ir \\ 12 & = V _ {\ text {терминал}} + 6 (\ text {0,6}) \\ \ поэтому V _ {\ text {terminal}} & = \ text {8,4} \ text {V} \ end {align *} \ begin {align *} I _ {\ text {2,4} \ \ Omega} & = \ frac {V} {R} \\ & = \ frac {\ text {8,4}} {\ text {2,4}} \\ & = \ text {3,5 A} \ end {align *} \ begin {align *} I _ {\ text {задние фонари}} & = 6 — \ text {3,5} \\ & = \ текст {2,5} \ текст {A} \\ R _ {\ text {задние фонари}} & = \ frac {V} {I} \\ & = \ frac {\ text {8,4}} {\ text {2,5}} \\ & = \ текст {3,36} \ \ Omega \\ R _ {\ text {задний фонарь}} & = \ text {1,68} \ \ Omega \ end {выровнять *}

Вариант 3:

\ begin {align *} V & = IR \\ \ text {12} & = \ text {6} (R) \\ R _ {\ text {ext}} & = 2 \ \ Omega \ end {align *} \ begin {align *} R _ {\ text {parallel}} & = 2 — \ text {0,6} \\ & = \ текст {1,4} \ \ Omega \\ \ frac {1} {R} & = \ frac {1} {R_ {1}} + \ frac {1} {R_ {2}} \\ \ frac {1} {\ text {1,4}} & = \ frac {1} {\ text {2,4}} + \ frac {1} {R} \\ R & = \ текст {3,36} \ \ Omega \ end {выровнять *}

Каждый задний фонарь: \ (R = \ text {1,68} \ \ Omega \)

Вариант 4:

Для параллельной комбинации: \ (I_ {1} + I_ {2} = 6 \ text {A} \)

\ begin {align *} \ поэтому \ frac {V} {\ text {2,4}} + \ frac {V} {R _ {\ text {задние фонари}}} & = \ text {6} \\ \ text {8,4} \ left (\ frac {1} {\ text {2,4}} + \ frac {1} {R _ {\ text {задние фонари}}} \ right) & = \ text {6 } \\ \ поэтому R _ {\ text {задние фонари}} & = \ text {3,36} \ \ Omega \\ R _ {\ text {задний фонарь}} & = \ text {1,68} \ \ Omega \ end {выровнять *}

(5 баллов)

Вопрос 4

Увеличивается

Сопротивление увеличивается, а ток уменьшается.

Разное

Добавить комментарий

Ваш адрес email не будет опубликован. Обязательные поля помечены *